Módulo 1 Med. Interna Flashcards

1
Q

A 23-year-old woman comes to the physician for a health maintenance examination. She enjoys good health and exercises regularly. Her height is 172 cm (68 in) and weight is 66 kg (145 lb). Her blood pressure is 120/80 mm Hg, pulse is 74/min, and respirations are 12/min. Physical examination is unremarkable except for heart auscultation, which reveals an isolated midsystolic click. Which of the following is the most common cause of this auscultatory finding?

(A) Bicuspid aortic valve
(B) Congenital pulmonary stenosis
(C) Mitral valve prolapse
(D) Ruptured papillary muscle
(E) Tricuspid regurgitation

A

Respuesta: C

The correct answer is C. The most characteristic manifestation of a floppy mitral valve is a midsystolic click. This frequently asymptomatic condition may be associated with chest pain, dyspnea, palpitations, and other nonspecific symptoms. Patients with a midsystolic click as the only sign are usually asymptomatic; however, those with a systolic murmur may have hemodynamically significant mitral valve regurgitation. In addition, mitral valve prolapse is associated with an increased incidence of infective endocarditis, arrhythmias, sudden death, and cerebral embolism.

Bicuspid aortic valve (choice A) is the most frequent type of congenital defect of the aortic valve. It may manifest with valvular stenosis, giving rise to a systolic murmur sometimes associated with an opening click.

Congenital pulmonary stenosis (choice B) is a rare condition that gives auscultatory signs similar to aortic stenosis, i.e., a harsh systolic click sometimes associated with an opening click.

A ruptured papillary muscle (choice D) may develop as a complication of infective endocarditis or myocardial infarction. It may lead to mitral or tricuspid regurgitation and thus manifest with a systolic murmur not associated with clicks.

Tricuspid regurgitation (choice E) manifests with a harsh systolic murmur that increases in intensity during inspiration. The most common cause is right ventricular overload; less common causes are infective endocarditis and right ventricular myocardial infarction.

How well did you know this?
1
Not at all
2
3
4
5
Perfectly
2
Q

A 50-year-old man comes to the physician because of gingival bleeding, epistaxis, and fever for 2 days. He appears acutely ill. His temperature is 39 C (102 F), blood pressure is 120/70 mm Hg, pulse is 120/min, and respirations are 22/min. Bilateral rhonchi are heard on chest examination. He is admitted for further evaluation. Chest x- ray shows bibasilar infiltrates consistent with bronchopneumonia. Blood tests show 12,000 leukocytes/mm3 with numerous myeloid blasts. Platelet count is 15,000/mm3. A bone marrow biopsy demonstrates hypercellular marrow, with 35% blasts. Elongated cytoplasmic inclusions consistent with Auer rods are appreciated in peripheral and marrow blasts. Which of the following is the most likely diagnosis?

(A) Acute lymphocytic leukemia (ALL)
(B) Acute myelogenous leukemia (AML) (C) Chronic myelogenous leukemia (CML)
(D) Leukemoid reaction
(E) Myelodysplastic

A

Respuesta: B

The correct answer is B. The clinical manifestations are consistent with acute myelogenous leukemia (AML). This disease of middle- aged people (median age at presentation is 50 years) is due to neoplastic transformation of a bone marrow stem cell that is incapable of differentiating into mature leukocytes. A large number of blasts invade the bone marrow and the peripheral blood. AML is subdivided into seven types with different prognostic and therapeutic implications. These types are determined on the basis of the degree of maturation of blasts, their morphology, and coexisting cytogenetic abnormalities. Leukocytosis may be mild or even absent (“aleukemic leukemia”). However, most of the circulating leukocytes are blasts or immature myeloid forms. Signs and symptoms result from neutropenia, anemia, and thrombocytopenia. Auer rods consist of eosinophilic, needle-like inclusions in myeloid cells and are pathognomonic of AML.

In contrast to AML, acute lymphocytic leukemia (ALL; choice A) is a disease of children, with a peak of incidence from 3 to 7 years. ALL represents 80% of all cases of acute leukemia in children. The clinical presentation is similar to that of AML. The lymphocytic nature of blasts is confirmed by demonstrating lymphocytic markers, such as terminal deoxynucleotide transferase (TdT).

Chronic myelogenous leukemia (CML; choice C) is also a disorder of middle-aged persons. Neoplastic bone marrow precursors in this condition are still capable of differentiating along myeloid lines, so that most circulating leukemic cells appear as mature white blood cells. CML is a myeloproliferative disorder, so the platelet count and erythrocyte count are usually normal or even increased. Leukocytosis in CML is usually striking, often higher than 500,000/mm3. The Philadelphia chromosome, the result of a balanced translocation between 9q and 22q, is present in 95% of cases.

A leukemoid reaction (choice D) is defined as an abnormal elevation of the white cell count in response to infections. Circulating leukocytes, however, are normal in morphology and never exceed 50,000/mm3. Leukocyte alkaline phosphatase is useful in differentiating leukemoid reaction from myeloid leukemia. The enzyme is elevated in leukemoid reaction but low in leukemia.

Myelodysplastic syndromes (choice E) constitute a complex set of bone marrow disorders, in which at least two cell lines are affected. These conditions are characterized by cytopenias (anemia, thrombocytopenia, and/or neutropenia) associated with hypercellular bone marrow. Cytopenias are due to ineffective hematopoiesis. The progression is usually indolent, but transformation to acute leukemia occurs in some cases.

How well did you know this?
1
Not at all
2
3
4
5
Perfectly
3
Q

A previously healthy 30-year-old man is injured in an automobile accident. He is taken to the emergency department, where he is noted to have multiple lacerations of his extremities, some of which are bleeding profusely. His blood pressure is 70/palpable mm Hg. The decision is made to transfuse 2 units of blood after rapid cross- matching. No reactions are detected in the blood bank. Ten minutes after the transfusion, the patient develops a severe case of hives. The development of hives in this setting would be most likely to be seen in a patient with which of the following syndromes?

(A) Adenosine deaminase deficiency (B) Ataxia telangiectasia
(C) DiGeorge syndrome
(D) Selective IgA deficiency
(E) Wiskott-Aldrich syndrome

A

Respuesta: D

The correct answer is D. Selective IgA deficiency is a relatively common condition (1 in 700 incidence in Caucasians), in which patients are genetically unable to synthesize IgA for either serum or bodily secretions. The underlying defect is a failure of B cells to differentiate into IgA-producing plasma cells. Most individuals with selective IgA deficiency are asymptomatic; about 5% have recurrent respiratory tract infections. The condition becomes clinically significant when blood transfusion is required, since they may develop anaphylaxis when exposed to blood products containing IgA. This can happen even on the first transfusion, presumably because they have been exposed to IgA in animal products that they have eaten. The condition can be confirmed with serum electrophoresis studies, which show an absence of IgA. Once diagnosed, the individuals (and families) need to be taught to tell their physicians about the IgA deficiency, so that only IgA free transfusions will be used.

Adenosine deaminase deficiency (choice A) is a cause of severe combined immunodeficiency.

Ataxia telangiectasia (choice B) is characterized by cerebellar ataxia, telangiectasias, and immunodeficiency.

DiGeorge syndrome (choice C) is characterized by hypoparathyroidism, thymic aplasia, and deficient T cell function.

Wiskott-Aldrich syndrome (choice E) is characterized by thrombocytopenia, lymphopenia, and atopic eczema.

How well did you know this?
1
Not at all
2
3
4
5
Perfectly
3
Q

A 48-year-old man comes to the physician because of a 2-day history of severe low back pain. He states that he has had periodic low back pain for years, but this is more severe than usual and radiates to the buttock and down the right leg. His temperature is 36.8 C (98.2 F). Examination shows some rigidity of the lumbar spine. The pain is exacerbated by applying pressure on the paravertebral region in the lower lumbar spine and by passively raising the leg at 45 degrees while the patient lies supine. A reduced Achilles tendon reflex is noted. Which of the following is the most appropriate next step in management?

(A) MRI examination of vertebral column
(B) Nonsteroidal anti-inflammatory drugs (NSAIDs) and 2 days of bed rest
(C) Plain x-ray examination of the lumbosacral spine
(D) Radionuclide bone scanning
(E) Surgical consultation

A

Respuesta: B

The correct answer is B. The clinical picture strongly suggests herniation of an intervertebral disc causing compression of a spinal root (S1, considering radiation of the pain and reflex alterations). Supporting such a diagnosis is also the positive straight leg-raising test (Lasegue sign). When the history and physical examination support a diagnosis of disc herniation, conservative management is all that is needed. Current recommendations include treatment with NSAIDs and bed rest of short duration (no longer than 2 days). Longer periods of bed rest do not provide any additional benefit.

MRI examination of vertebral column (choice A) is certainly the diagnostic procedure of choice to visualize soft tissue structures of the vertebral column. MRI is reserved for cases in which more detailed imaging information would change the therapeutic approach.

Plain x-ray examination of the lumbosacral spine (choice C) provides nonspecific information. Almost any person older than 40 has some signs of degenerative joint disease of the lumbar column. Plain radiographs should be performed when the clinical symptomatology suggests diseases other than disc herniation, such as tumors or infections.

Radionuclide bone scanning (choice D) is useful in detecting foci of osteomyelitis or bone metastases, but not disc disease.

Surgical consultation (choice E) should be sought if the patient does not respond to appropriate treatment or if there are severe or evolving neurologic deficits. Percutaneous lumbar discectomy may be performed under local anesthesia as an alternative to laminectomy.

How well did you know this?
1
Not at all
2
3
4
5
Perfectly
4
Q

A 71-year-old man presents with a complaint of dyspnea for the past week. The patient has a history of diabetes and hypertension and was recently diagnosed with cancer. He is currently on multiple drug therapy. On examination, temperature is 37.2 C (99.0 F), his blood pressure is 140/90 mm Hg, pulse is 90/min, and respirations are 22/min. His lungs have a few crackles at the bases with no wheezing. A chest x-ray film shows bilateral diffuse interstitial markings. Which of the following medications is likely responsible for the patient’s dyspnea?

(A) Bleomycin
(B) Cisplatin
(C) Mithramycin
(D) Verapamil
(E) Vincristine

A

Respuesta: A

The correct answer is A. Bleomycin can cause pneumonitis, which progresses to pulmonary fibrosis. Cough and shortness of breath are indications of the development of this complication. The pulmonary toxicity is usually age-and dose-related, although an allergic form of pneumonitis has been reported.

Cisplatin (choice B) produces tinnitus, hearing loss, and nephrotoxicity.

Mithramycin (choice C) is associated with hemolytic uremic syndrome, thrombocytopenia, hepatotoxicity, and nephrotoxicity.

Calcium-channel blockers, such as verapamil (choice D), cause headache, dizziness, and nausea. They may increase the extent of heart block and worsen congestive heart failure.

Vincristine (choice E) causes neuropathy rather than pulmonary symptoms.

How well did you know this?
1
Not at all
2
3
4
5
Perfectly
5
Q

A 30-year-old woman complains of fatigue and dyspnea for the past 2 months. She reports that she has also lost 15 pounds during this time. She has been previously healthy and is not taking any medications. She is pale and thin and has a flow murmur on her cardiac examination. She also has mildly enlarged supraclavicular lymph nodes. Laboratory results are notable for a hematocrit of 30%, mean corpuscular volume (MCV) of 78 μm3, decreased transferrin iron binding capacity (TIBC), and increased ferritin. A screening erythrocyte protoporphyrin level is <35 μg/dL, and a blood smear shows microcytic red cells. Which of the following is the most likely diagnosis?

(A) Anemia of chronic disease
(B) Aplastic anemia
(C) Lead poisoning
(D) Pyridoxine deficiency
(E) Spherocytosis
(F) Thiamine deficiency

A

Respuesta: A

The correct answer is A. This is anemia of chronic disease. In contrast to iron deficiency anemia, the TIBC is decreased, but ferritin is increased. The microcytosis is the same as in iron deficiency. This patient may have an infection or an occult malignancy and needs further workup for the loss of weight.

Aplastic anemia (choice B) can be the result of exposure to drugs, such as chloramphenicol, that can lead to the suppression of erythrocyte production. A low reticulocyte count, which is an indication of immature red cells, can be helpful in diagnosis of this illness. Bone marrow biopsy may show hypocellularity of the marrow.

Lead poisoning (choice C) occurs following the inhalation of lead dust or fumes or following the ingestion of lead. Presentation in these patients ranges from abdominal discomfort, myalgia, headache, and weight loss to peripheral neuropathy and encephalopathy. Laboratory studies show a normal serum iron, a normal TIBC, and basophilic stippling on peripheral blood smear. A screening erythrocyte protoporphyrin level is >35 μg/dL, indicating the need for blood lead testing.

Pyridoxine is a cofactor in the manufacture of porphyrins, which are needed in the manufacture of hemoglobin. Therefore, pyridoxine deficiency (choice D) may cause a microcytic hypochromic anemia, but the laboratory panel above is characteristic of anemia of chronic disease.

Hereditary spherocytosis (choice E) is a genetic defect arising from mutations in red cell cytoskeletal proteins. This leads to a cell wall defect, which in turn leads to removal of excess cell wall in the spleen. Surface tension causes these cells to become spheres.

Thiamine deficiency (choice F) has been implicated in a rare megaloblastic anemia in children. It is also seen in malnourished alcoholics, and intramuscular repletion is often required.

How well did you know this?
1
Not at all
2
3
4
5
Perfectly
6
Q

A 50-year-old man comes to the physician because of an unusual appearing mole on his upper back. He says that his wife has noted a recent change in its color and shape. The lesion measures 0.7 cm and has ill-defined margins and irregular pigmentation. The patient is otherwise healthy and takes no medication. Which of the following is the most appropriate next step in management?

(A) Follow-up examination in 6 months
(B) Topical application of Podophyllum resin
(C) Cryotherapy with liquid nitrogen
(D) Shave biopsy
(E) Incisional biopsy
(F) Excisional biopsy

A

Respuesta: F

The correct answer is F. The gross appearance of the lesion, along with its recent changes over a presumably short period, is highly suggestive of malignant melanoma. The proportion in which melanomas arise from pre-existing benign nevocellular nevi is not known. In the dysplastic nevus syndrome, however, a dysplastic nevus-melanoma sequence is well established. Nevertheless, an excisional biopsy should be carried out in any case of pigmented skin lesion that shows one or more of the following features: asymmetric or fuzzy border, irregular or variegated color, and diameter greater than 0.6 cm. According to the American Cancer Society, the mnemonic ABCD may serve to recall the most important suspicious signs: Asymmetry, Border irregularity, Color variegation, and Diameter >0.6 cm. Bleeding and ulcerations are malignant signs, albeit far less frequent. Melanoma is the most common cause of death due to skin malignancies. Physicians can play a crucial role in prevention by referring to dermatologists patients who have moles with such suspicious features. The initial approach to a suspicious mole or clinically obvious melanoma consists of total excision (excisional biopsy) with a small margin. If a diagnosis of melanoma is confirmed pathologically, wider margins are excised on a second operation.

Follow-up examination in 6 months (choice A) would result in a dangerous delay in diagnosis and treatment.

Topical application of Podophyllum resin (choice B) and cryotherapy with liquid nitrogen (choice C) are treatments used for common and genital warts, as well as for other common benign lesions, such as seborrheic keratosis. These methods should never be used on pigmented lesions.

Shave biopsy (choice D) is applicable to many types of superficial skin lesions, including basal cell carcinomas, but is inappropriate for melanomas. Proper diagnosis and evaluation of depth of invasion in melanomas can be achieved only on full-thickness biopsies.

Incisional biopsy (i.e., partial sampling; choice E) is not appropriate unless the lesion is too extensive (such as giant congenital nevi or lentigo maligna). However, there seems to be no foundation for the belief that incisional biopsy facilitates cancer spread.

How well did you know this?
1
Not at all
2
3
4
5
Perfectly
7
Q

A 52-year-old man with a history of chronic low back pain complains of 3 days of a cough productive of purulent sputum, fever, and left-sided subcostal pain worsened by breathing. A single episode of shaking chills accompanied the onset of the illness. He has no gastrointestinal complaints. His temperature is 40 C (104 F), blood pressure is 160/80 mm Hg, pulse is 100/min, and respirations are 38/min with nasal flaring and splinting. The cardiac and abdominal examinations are within normal limits. There are moist crackles and egophony at the left lung base. A chest x-ray film shows a left lower lobe infiltrate. Gram stain of the sputum shows multiple polymorphonuclear leukocytes and occasional epithelial cells. Which of the following is the most likely pathogen?

(A) Gram-negative diplococci
(B) Gram-negative rods
(C) Gram-positive cocci in clusters
(D) Gram-positive diplococci in chains
(E) Gram-positive rods

A

Respuestas: D

The correct answer is D. This patient is demonstrating the classic picture of pneumococcal pneumonia. Streptococcus pneumoniae is the most common cause of community-acquired pneumonia in this age group. The usual presentation is sudden onset of shaking chills, with rigors, high fever, and difficulty breathing. Pleuritic chest pain is often present and signifies bacterial infection. A white blood cell count, not provided in this case, most often is significantly elevated with a left shift (predominantly bands and polymorphonuclear cells). Chest x-ray films usually reveal a lobar distribution of the pneumonia. Pleural effusions are present in up to 30% of the cases. Gram stain of the sputum commonly reveals gram-positive diplococci in chains.

Gram-negative diplococci (choice A) would be present in pneumonia due to Moraxella catarrhalis (formerly Branhamella catarrhalis). This pathogen may produce acute pneumonia and usually occurs in the elderly or in those with a history of chronic bronchitis or obstructive lung disease.

Gram-negative rods (choice B) are not a usual cause of pneumonia in this population of patients. Gram-negative rods causing pneumonia include Klebsiella, Enterobacter, Serratia, and Proteus, which occur more commonly in patients who are debilitated or residing in nursing homes or similar institutions. These bacteria are often responsible for nosocomial pneumonias and, infrequently, community-acquired pneumonia.

Gram-positive cocci in clusters (choice C) that cause pneumonia are usually Staphylococcus aureus. S. aureus is an uncommon cause of community-acquired pneumonia. When it does cause disease, it is usually during or just following an epidemic of viral influenza. S. aureus may be seen year-round in the hospital, because it is a common cause of nosocomial pneumonia.

Gram-positive rods (choice E) would likely be Corynebacterium diphtheriae (diphtheria). This patient presents with pneumonia, not diphtheria (an infection that occurs in the pharynx, middle ear, larynx, skin, or bronchi).

How well did you know this?
1
Not at all
2
3
4
5
Perfectly
8
Q

A 38-year-old man who works as a reporter for a travel magazine comes to his physician because of the acute onset of jaundice, malaise, and temperatures to 38.5 C (101 F). He had returned from Burma 2 weeks ago, where he spent 4 weeks. He says that he abstains from alcohol beverages and does not take any medications. Laboratory studies show elevated serum aminotransferases, high bilirubin (both total and direct), and negative serology for hepatitis A virus (HAV) and C virus (HCV) infection. He was vaccinated for hepatitis B virus (HBV) 3 years ago and is now positive for anti- HBsAg antibodies. Which of the following serologic markers should be tested as the most appropriate next step in diagnosis?

(A) Anti-HCV IgG antibodies by RIBA
(B) Anti-HDV IgG antibodies
(C) Anti-HEV IgM antibodies
(D) Anti-HGV IgG antibodies
(E) HBsAg

A

Respuesta: C

The correct answer is C. Hepatitis E virus (HEV) is rare in the U.S., but outbreaks of acute hepatitis E occur in some countries, namely Mexico, India, Afghanistan, and Burma. The infectious agent is transmitted by the oralfecal route, usually from infected water. HEV hepatitis is generally self-limited, but it is important to note that it carries a 10% to 20% mortality rate in pregnant women. Similar to HAV, HEV infection does not cause chronic hepatitis or a carrier state. The onset of IgM antibodies to HEV is concomitant with the appearance of clinical symptomatology, whereas IgG will become detectable after the acute phase.

RIBA (recombinant immunoblot assay) for anti-HCV antibodies (choice A) is used to confirm a diagnosis of hepatitis C in patients with positive anti-HCV antibodies by the more conventional enzyme immunoassay test. The latter has a 50% specificity, and false positive results are likely in patients with hypergammaglobulinemia. Occasionally, HCV RIBA is also used when the enzyme immunoassay is negative, but there are strong clinical grounds to suspect that the patient has HCV hepatitis. There is no reason to suspect HCV hepatitis in this case.

Anti-HDV antibodies (choice B) are found in association with hepatitis D virus infection. HDV is a “defective” RNA virus that causes hepatitis only in the presence of the surface antigen of HBV (HBsAg). In the U.S., HDV is usually associated with IV drug abuse. HDV may coinfect or superinfect patients with HBV. Superinfection is associated with increased risk for fulminant hepatitis or rapid progression to cirrhosis.

Anti-HGV antibodies (choice D) are found in 50% of IV drug abusers and 30% of patients receiving hemodialysis. HGV is a flavivirus transmitted by the parenteral route. The infection is followed by viremia lasting for at least 10 years. The pathogenic role of HGV is still uncertain.

HBsAg (choice E) detection implies ongoing HBV replication in the organism. HBsAg appears first in the blood before HBV infection becomes clinically evident. Persistence of HBsAg is associated with infectivity. HBV infection would be most unlikely in this case, since the patient developed anti-HBV antibodies after vaccination.

How well did you know this?
1
Not at all
2
3
4
5
Perfectly
9
Q

A 50-year-old man returns to his home in Minnesota after a diving trip to Belize (Central America). The day after his return, he comes to the physician because of diarrhea, abdominal cramps, and nausea. His temperature is 37 C (98.6 F). His stools do not contain mucus or blood. Microscopic examination of a stool sample reveals no leukocytes. Which of the following is the most likely pathogen?

(A) Bacillus cereus
(B) Clostridium perfringens
(C) Escherichia coli
(D) Rotavirus
(E) Staphylococcus aureus

A

Respuesta: C

The correct answer is C. Traveling abroad often entails abrupt changes in diet and climate, as well as exposure to conditions of poor sanitation, all of which results in a high incidence of diarrhea. This is self-limiting and manifests with watery diarrhea and dehydration, but no fever or other signs of systemic infection. The most frequent cause of traveler’s diarrhea is enterotoxigenic Escherichia coli.

The remaining infectious agents listed here are all potential causes of noninflammatory diarrhea, which is not associated with blood and mucus in the stool, fever, systemic signs of infection, or fecal leukocytes.

Bacillus cereus (choice A), Clostridium perfringens (choice B), and Staphylococcus aureus (choice E), along with enterotoxigenic E. coli, are the most common agents associated with food poisoning due to production of toxins. All these pathogens produce a similar clinical picture of watery diarrhea, sometimes with nausea and vomiting, but no fever.

Rotavirus (choice D) is one of the most important infectious causes of diarrhea in infants and young children in developing countries. It may also cause diarrhea in adults exposed to infected children.

How well did you know this?
1
Not at all
2
3
4
5
Perfectly
10
Q

A 65-year-old man with a history of peripheral vascular disease develops thromboembolic disease in his left leg accompanied by dry gangrene. Laboratory tests show elevated serum lactic acid, and his arterial pH is 7.27. An ECG in this patient is most likely to show which of the following?

(A) Peaked T waves
(B) QT prolongation
(C) ST depression
(D) T wave inversion
(E) U waves

A

Respuesta A

The correct answer is A. Peaked T waves are associated with significant hyperkalemia that may lead to arrhythmia. In this patient, the primary mechanism of hyperkalemia is acidosis. As a result of the lowered pH, the extracellular concentration of protons increases, thereby increasing the H+/K+ antiports on the cell surface, driving protons into the cells and potassium into the extracellular space. ECG changes are an indication for immediate correction of the hyperkalemia, indicating an increased risk of arrhythmia. Calcium gluconate should be administered to decrease membrane excitability.

Hypocalcemia causes prolonged QT intervals (choice B). The QT interval is the time difference between ventricular depolarization and repolarization. Since the QT interval depends on the heart rate, the corrected QT interval (QTc) is often used. The correction factor incorporates the interval between consecutive P waves.

ST depression (choice C) would be seen in an ischemic event. It is important to compare the new ECG with an old one to determine whether the depression is new. If this is the case, the patient with such ECG changes should at least be placed on aspirin and observed for an ischemic event.

T wave inversion (choice D) is another indication that the patient may be undergoing an ischemic event. Once again, it is important to compare the new ECG with an old one. Furthermore, if the new ECG shows upright T waves, but the old one shows inverted T waves, this denotes “pseudonormalization” and once again indicates an ischemic event.

U waves (choice E) are seen in hypokalemia. If an ECG shows these changes, the risk of an arrhythmia is significant, and the hypokalemia must be corrected immediately. This can usually be achieved by administering oral potassium, but occasionally IV potassium may be required.

How well did you know this?
1
Not at all
2
3
4
5
Perfectly
11
Q

A 35-year-old man has had nocturnal attacks of severe periorbital headache for the past 5 days. Each episode awakens him at night within 2 hours of falling asleep, lasts for less than an hour, and is associated with ipsilateral rhinorrhea and lacrimation. There is no family history of similar headaches. Careful evaluation does not reveal any objective evidence of neurologic dysfunction. The pupils are equal and normally reactive to light. His temperature is 37 C (98.6 F), blood pressure is 125/75 mm Hg, and pulse is 72/min. Which of the following is the most likely diagnosis?

(A) Cluster headache
(B) Depression headache
(C) Giant cell arteritis
(D) Migraine
(E) Tension headache
(F) Trigeminal neuralgia

A

Respuesta: A

The correct answer is A. The clinical presentation is characteristic of cluster headache. In its classic form, cluster headache manifests as nocturnal attacks that last between 30 minutes and 2 hours. These are often precipitated by alcohol consumption and recur daily for up to 8 weeks. Each “cluster” is then followed by a pain-free interval lasting for 1 year on average. The pathogenesis is probably related to disturbances of the serotoninergic pathways originating from the raphe nuclei. Acute attacks may be shortened by oxygen, sumatriptan, and ergotamine preparations; several prophylactic agents are available to prevent clusters.

Depression headache (choice B) is often worse in the morning and is frequently associated with other manifestations of depression.

The headache due to giant cell arteritis (choice C) usually manifests in elderly patients and is associated with scalp tenderness over the affected superficial temporal artery. Systemic signs and symptoms are present, including myalgia, weight loss, and malaise. The erythrocyte sedimentation rate is elevated.

Classic cases of migraine (choice D) begin in early adulthood and manifest as episodic unilateral throbbing headache, often associated with nausea, photophobia, and visual symptoms.

Tension headache (choice E) has a diffuse, band-like character and feels worse in the back of the head. Pain slowly increases and may last for many hours or even days.

Trigeminal neuralgia (choice F) is a disorder of the sensory nucleus of CN V that produces episodic, severe, and lancinating pain in the distribution of one or more divisions of the trigeminal nerve. Pain is often precipitated by well defined trigger zones (e.g., washing or shaving) and is not associated with Horner syndrome or rhinorrhea.

How well did you know this?
1
Not at all
2
3
4
5
Perfectly
12
Q

A 32-year-old African American woman complains of mild fevers and fatigue for the past month. She has no significant past medical history. Her temperature is 38.1 C (100.6 F), blood pressure is 115/70 mm Hg, pulse is 75/min, and respirations are 18/min. Nontender, mobile, cervical and axillary lymph nodes are noted. Auscultation of the lungs reveals fine crackles bilaterally. A chest x- ray film shows hilar lymphadenopathy and diffuse interstitial infiltrates. Lymph node biopsy shows noncaseating granulomas. Which of the following is the most appropriate therapy?

(A) Allopurinol
(B) Angiotensin converting enzyme (ACE) inhibitor
(C) Cyclosporine
(D) Glucocorticoids
(E) Isoniazid

A

Respuesta: D

The correct answer is D. This patient has pulmonary sarcoidosis. The peak age group for sarcoidosis is 20-40 years, and the disease seems to be more common in blacks. Noncaseating granulomas can occur in the lungs, heart, kidneys, skin, liver, or other organs. Most characteristically, the patients are asymptomatic, and the disease is detected by an abnormal chest x-ray film, which usually shows bilateral symmetric hilar adenopathy often associated with paratracheal adenopathy and/or parenchymal infiltrates. Patients may have uveitis, peripheral arthritis, skin involvement with granulomas, or erythema nodosum. The lungs are the most frequently involved organ; pulmonary symptoms, when present, include dyspnea on exertion, nonproductive cough, and wheezing. Radiologic abnormalities are graded 0-3. Grade 0 is associated with a normal x-ray. Grade 1 is associated with lymph node enlargement without pulmonary parenchymal abnormalities. Grade 2A is a combination of lymph node and diffuse pulmonary parenchymal disease. Grade 2B is a diffuse parenchymal disease without lymph node enlargement. Grade 3 is associated with radiographic changes indicating more chronic disease with pulmonary fibrosis (“honey- combing”). Many patients show spontaneous total remission of disease for a period up to 3 years. Prednisone is usually the drug of choice for treatment, with a starting dose of 30-40 mg/day.

Neither allopurinol (choice A) nor cyclosporine (choice C), an immune modulator, has been proven to be of benefit in sarcoidosis.

Levels of ACE may be elevated in patients with sarcoidosis but are also elevated in many other diseases. This enzyme elevation is thought to be related to induction by the granulomas. There is no evidence that ACE inhibitors (choice B) have any therapeutic value in treatment of sarcoidosis. At one point in history, some theorized that sarcoidosis was caused by a type of mycobacterium, related to tuberculosis. However, this has not been definitively proven. Furthermore, isoniazid (choice E) has not been shown to be beneficial.

How well did you know this?
1
Not at all
2
3
4
5
Perfectly
13
Q

A 20-year-old man comes to the physician because he has noticed blood in his urine on several occasions in the past year. Each episode of hematuria occurred in association with an upper respiratory tract infection or a flulike illness. Physical examination is unremarkable. A urine dipstick test shows mild proteinuria and microhematuria. Serum levels of electrolytes, creatinine, and blood urea nitrogen are within normal limits. Serum levels of IgA are elevated. Which of the following is the most likely diagnosis?

(A) Berger disease
(B) Goodpasture syndrome
(C) Henoch-Schönlein purpura
(D) Minimal change disease
(E) Postinfectious glomerulonephritis
(F) Wegener granulomatosis

A

Respuesta: A

The correct answer is A. The clinical presentation is consistent with Berger disease or IgA nephropathy, the most frequent form of glomerulonephritis in the U.S. (and probably worldwide). Often, microhematuria or mild proteinuria occurs as an incidental finding or as recurring episodes following upper respiratory or intestinal infections. IgA deposition in the mesangium is the most characteristic morphologic abnormality, and serum IgA is increased in 50% of patients (hence the designation). Up to 50% of patients will eventually progress to chronic renal failure.

Goodpasture syndrome (choice B) typically involves both the lungs and kidneys. Hemoptysis and nephritic syndrome are the clinical manifestations. Linear deposition of anti-collagen antibodies along the glomerular and pulmonary basement membranes is the pathognomonic finding on biopsy. This is a severe condition that requires aggressive immunosuppressive treatment.

The renal changes of Henoch-Schönlein purpura (choice C) are very similar to those of Berger disease. These conditions, in fact, are thought to represent different manifestations of a common spectrum of diseases, in which autoimmune damage is mediated by IgA. In Henoch-Schönlein purpura, nephritic syndrome is associated with palpable purpura, arthralgias, and abdominal pain. The disorder usually affects children.

Minimal change disease (choice D) is characterized by edema, albuminuria, and changes in blood lipids and proteins. It is usually seen in children, and it doesn’t present with episodic hematuria. Proteinuria is of the nephrotic range.

Postinfectious glomerulonephritis (choice E) commonly occurs 1-2 weeks after an infection by group A Streptococcus (pharyngitis or impetigo) and manifests with nephritic syndrome. Hematuria developing in the setting of Berger disease, instead, is concomitant with an upper respiratory infection (so-called sympharyngitic hematuria).

Wegener granulomatosis (choice F) is a necrotizing granulomatous vasculitis involving the upper respiratory system and the kidneys. Systemic symptoms are present, with fever, weight loss, and malaise. Aggressive immunosuppression is the mainstay of therapy.

How well did you know this?
1
Not at all
2
3
4
5
Perfectly
14
Q

A 60-year-old woman presents to a physician complaining of a swelling in her neck. Her past medical history is significant for rheumatoid arthritis and Sjögren syndrome. Physical examination reveals a mildly nodular, firm, rubbery goiter. Total serum thyroxine (T4) is 10 mg/dL, and third-generation thyroid-stimulating hormone (TSH) testing shows a level of 1.2 mIU/mL. Antithyroid peroxidase antibody titers are high. Which of the following is the most likely diagnosis?

(A) Euthyroid sick syndrome
(B) Graves disease
(C) Hashimoto thyroiditis
(D) Silent lymphocytic thyroiditis
(E) Subacute thyroiditis

A

Respuesta: C

The correct answer is C. Hashimoto disease is a chronic, destructive lymphocytic infiltration of the thyroid glands. It is probably the most common cause of primary hypothyroidism in the U.S. It has an 8:1 female predominance and increases in incidence with age. Many patients also have other autoimmune diseases. The description of the goiter in the question stem is typical of that produced by Hashimoto disease; the physical signs and symptoms of hypothyroidism are also present in longer-standing cases. Early in the disease, as in this case, T4 and TSH levels may be normal. Antithyroid peroxidase antibodies (against the specific antigen formerly detected with antimicrosomal antibodies) are observed in almost all patients with Hashimoto disease, but can also sometimes be detected in patients with Graves disease and silent lymphocytic thyroiditis.

Euthyroid sick syndrome (choice A) occurs in patients with severe systemic illness who are clinically euthyroid but have abnormal thyroid function tests.

Graves disease (choice B) causes diffuse toxic goiter and would exhibit both the signs and laboratory findings of hyperthyroidism.

Silent lymphocytic thyroiditis (choice D) usually occurs in postpartum women and may be a mild, usually spontaneously reversible, variant of Hashimoto disease.

Subacute thyroiditis (choice E) is a virally caused acute inflammatory disease that causes thyroid tenderness and pain.

How well did you know this?
1
Not at all
2
3
4
5
Perfectly
15
Q

A 22-year-old man comes to the physician because he has developed patches of hair loss in his scalp and beard over several months. The patient’s medical history is unremarkable, but his family history is significant for Addison disease in a sister and vitiligo in his mother. Physical examination shows two sharply demarcated areas of total hair loss in the scalp and one in the right cheek. The skin in these areas is perfectly smooth and covered only by sparse short hair shafts. In addition, pitting of nail plates is noted. A biopsy of affected skin demonstrates lymphocytic infiltrates around hair follicles. Which of the following is the most appropriate next step in management?

(A) Psychiatric consultation
(B) Topical application of minoxidil
(C) Oral administration of iron sulfate
(D) Topical injections of corticosteroids
(E) Systemic corticosteroids

A

Respuesta: E

The correct answer is D. The skin condition described is alopecia areata, a form of baldness that may affect any region of the body. Its pathogenesis is probably immunerelated, considering the frequent association with autoimmune disorders, such as Hashimoto thyroiditis, pernicious anemia, and Addison disease. The finding of perifollicular lymphocytic infiltration in affected areas seems to support an autoimmune origin. The patches of hair loss in alopecia areata are rather haphazardly distributed but sharply demarcated. An important feature is the presence of “exclamation hairs” (tiny hair shafts) in the zone of active shedding. In 80% of cases, the hair regrows spontaneously, but permanent loss is observed in the other 20%. Local injection of triamcinolone or application of anthralin ointment has been beneficial in hastening recovery.

Psychiatric consultation (choice A) may be appropriate for trichotillomania, a compulsive habit of pulling one’s own hair. In this disorder, the patches of hair loss are often unilateral (on the right side if the patient is right-handed, and vice versa), have irregular borders, and consistently show growing hairs.

Topical application of minoxidil (choice B) is widely used for treatment of the most common form of alopecia, namely androgenetic baldness, which is related to androgenic hormonal influences. The pattern of androgenetic baldness is characteristic and well known. Minoxidil treatment results in temporary hair regrowth, especially in patients younger than 50 and those with less extensive areas of hair loss.

Oral administration of iron sulfate (choice C) may be beneficial in another form of alopecia, named telogen effluvium. In this condition, an increased number of hairs are lost daily on combing or shampooing. It is due to an increase in the percentage of hairs in telogen (resting) phase and occurs in association with severe malnutrition, termination of pregnancy, stress from surgery or infection, and oral contraceptives. However, some studies have indicated that iron deficiency may play an important causative role in telogen effluvium.

Systemic corticosteroids (choice E) have not been shown to have any advantage over local injection in cases of alopecia areata.

How well did you know this?
1
Not at all
2
3
4
5
Perfectly
16
Q

A 75-year-old white man complains to a physician of abdominal pain. His temperature is 37 C (98.6 F), blood pressure is 110/65 mm Hg, pulse is 63/min, and respirations are 16/min. The abdomen is soft, with focal tenderness in the left lower quadrant. His erythrocyte count is 4.5 million/mm3, leukocyte count is 9000/mm3 with 60% neutrophils and 5% bands, and platelet count is 250,000/mm3. Serum chemistries show:
Sodium Potassium Chloride Creatinine Urea nitrogen
140 mEq/L 5 mEq/L 102 mEq/L 1.1 mg/dL 12 mg/dL
Which of the following is the most appropriate next step in diagnosis?

(A) Barium enema
(B) Colonoscopy
(C) CT of abdomen
(D) Plain film of abdomen
(E) Trial therapy of a liquid diet

A

Respuesta: E

The correct answer is E. The probable diagnosis is diverticulitis. The relatively mild symptoms, normal vital signs, and normal laboratory values in this patient indicate that he is not very ill. This means that he can be treated at home with rest, a liquid diet, and oral antibiotics, such as cephalexin. In this setting, the response to the therapeutic trial itself serves as a confirmatory test. In most cases, the symptoms will resolve rapidly on this regimen, and the diet can be gradually advanced to a soft, low-roughage diet supplemented with daily psyllium seed extract. At approximately 1 month, a high- roughage diet is resumed. Seriously ill patients with diverticulitis are usually treated in the hospital and may require IV antibiotics. Surgery will be needed in only about 20% of these patients.

Barium enema studies (choice A), typically with air contrast, can be performed if necessary at 2 weeks to confirm the presence of diverticula.

Colonoscopy (choice B) can be used alternatively to barium enema at 2 weeks to identify diverticula and rule out other conditions, such as proctitis or colon cancer, which would not be as likely in this patient with mild symptoms.

CT of the abdomen (choice C) is used in severe cases when the differential diagnosis includes pelvic abscess and appendicitis.

Plain film of the abdomen (choice D) is of limited utility in this setting but may show increased gas in the bowel if the diverticulitis has lowered intestinal motility.

How well did you know this?
1
Not at all
2
3
4
5
Perfectly
17
Q

A 38-year-old man comes to the physician because of slowly progressive visual problems that make him “bump into objects” on both sides. He also reports that, while driving, he has trouble switching lanes because he needs to turn his head all the way backward to look for other cars. Ocular examination shows bitemporal field loss with preserved visual acuity. Examination of the fundus is unremarkable. Which of the following is the most likely diagnosis?
(A) Pituitary adenoma
(B) Occipital lobe meningioma
(C) Optic glioma
(D) Optic nerve atrophy
(E) Optic neuritis
(F) Retinal detachment

A

Respuesta: A

The correct answer is A. The visual deficit present in this patient is described as bilateral temporal hemianopia and is due to chiasmatic lesions that compromise the crossing fibers originating from the temporal retina. A large pituitary adenoma (macroadenoma) that extends beyond the sella turcica into the suprasellar region is the most common cause of temporal hemianopia. Craniopharyngioma and meningioma are other causes.

Occipital lobe meningioma (choice B) may push on the visual cortex and produce visual symptoms that are referred to the contralateral half of the visual field (homonymous hemianopia).

Optic glioma (choice C) is a tumor of glial origin, usually an astrocytoma, that develops within the optic nerve. Visual symptoms develop slowly and are referred to the ipsilateral eye.

Optic nerve atrophy (choice D) involves damage to the nerve from ischemia, inflammation, glaucoma, toxic substances, and trauma. Symptoms include diminished visual acuity, reduced visual fields, abnormal color vision, and poor pupillary response to light. The optic disc appears pale or white on ophthalmoscopy.

Optic neuritis (choice E) will result in unilateral visual loss that develops rapidly. Multiple sclerosis is an important cause of optic neuritis.

Retinal detachment (choice F) results in blurring of vision that affects only one eye. Myopia and cataract extraction are the two most common predisposing factors.

How well did you know this?
1
Not at all
2
3
4
5
Perfectly
18
Q

An 18-year-old woman complains of myalgias, a sore throat, and painful mouth sores for 3 days’ duration. Her temperature is 38.2 C (100.8 F), blood pressure is 110/80 mm Hg, pulse is 84/min, respirations are 15/min. Her gingiva are edematous and erythematous, and there are vesicles on her right upper and lower lips. Her pharynx is mildly erythematous but without exudate, and there is tender mobile cervical lymphadenopathy. Her breath is not fetid, and the dentition is normal. Which of the following is the most likely causal agent?

(A) Actinomyces israelii
(B) Coxsackie virus
(C) Herpes simplex virus 1
(D) Nocardia asteroides
(E) Streptococcus pyogenes

A

Respuesta: C

The correct answer is C. Gingivostomatitis and pharyngitis are the most frequent clinical manifestations of primary herpes simplex virus 1 (HSV-1) infection, and are most commonly seen in children and young adults. Clinical signs and symptoms include fever, malaise, myalgias, and cervical adenopathy. Common lesions may involve the hard and soft palate, gingiva, tongue, lips, and facial area. The lesions are classically vesicular with an erythematous base.

Actinomyces israelii (choice A) causes an indolent suppurative infection. These anaerobic actinomycetes are commensals of the gastrointestinal tract and mouth. The organism may invade via a break in the oral mucosa or via aspiration into the lung. Poor dental hygiene and dental abscesses predispose to cervicofacial lesions. Infection presents as a chronic suppurative lesion with adjacent tissue showing inflammation with fibrosis and draining sinuses. Myalgias and low-grade fevers are rare with facial actinomycetes.

Coxsackie virus (choice B) infection may result in herpangina, an exanthematous disease characterized by acute onset of fever and sore throat. Small vesicular lesions and white papules (lymph nodules) surrounded by a red halo are typically seen over the posterior half of the palate, pharynx, and tonsillar areas. Lip and facial lesions are rare.

Nocardia asteroides (choice D) are aerobic actinomycetes that cause disease most often in the lung, but also at any site of tissue trauma. Lesions produced by Nocardia show suppuration, necrosis, and abscess formation with sinus tracts draining purulent material.

Streptococcus pyogenes (choice E) causes streptococcal pharyngitis, with the highest incidence in children aged 5-15 years. Patients usually present with the sudden onset of sore throat, particularly with pain on swallowing. Associated symptoms include fever, malaise, headache, and anorexia. On examination, there is diffuse edema and erythema of the posterior pharynx. The tonsils, if present, are enlarged and erythematous with an exudate. The cervical nodes are tender and enlarged. Oral lesions are limited to the posterior pharynx.

How well did you know this?
1
Not at all
2
3
4
5
Perfectly
19
Q

A 35-year-old man complains to a physician of chronic vague gastric pain of several years’ duration. The pain is sometimes relieved by food. Serum immunoglobulin studies for IgG and IgA antibodies directed against Helicobacter pylori are strongly positive. Endoscopy with gastric antral biopsy demonstrates gastritis but no ulcerative lesions. H. pylori organisms are seen with special stains on the biopsy fragments. The patient is treated with a 1-week course of omeprazole (20 mg bid), plus clarithromycin and metronidazole (500 mg bid each). Which of the following is the most appropriate test to noninvasively determine whether the H. pylori has been eradicated?

(A) Culture of gastric biopsy
(B) Rapid urease test
(C) Repeat qualitative IgA and IgG antibodies against H. pylori
(D) Repeat quantitative IgA and IgG antibodies against H. pylori
(E) Urea breath test

A

Respuesta: E

The correct answer is E. Helicobacter pylori is a small, gram- negative bacterium that lives in and locally destroys the mucus coating that lines the stomach. The organism has been linked to a wide variety of problems, including gastritis, peptic ulcer disease, gastric cancer, and gastric lymphoma. Because of these associations, physicians are tending to become more aggressive about therapy. H. pylori is a hardy organism and requires concurrent therapy with multiple agents for eradication. The original regimen combined bismuth subsalicylate (Pepto-Bismol), tetracycline, and metronidazole, and had only an 80% cure rate in compliant patients who followed the regimen for 2 weeks. The schedule listed in the question stem is a more modern one; it is both easier to follow and has a better cure rate. Alternative regimens may substitute amoxicillin (1 g bid) for metronidazole, or may substitute lansoprazole (30 mg bid) for omeprazole. These more effective regimens have caused a problem in determining whether eradication has occurred, however, because the course is so short that IgG and IgA antibodies against H. pylori have not had time to decrease by the end of therapy. The urea breath test is a relatively new test in which the patient is given oral urea that has been labeled with 13C or 14C. The H. pylori bacteria contain the enzyme urease and are able to metabolize the urea, producing radioactively labeled CO2, which can be measured in breath samples taken 20-30 minutes after ingestion. It is recommended that this test be delayed until 4 weeks after the end of the regimen, since recent antibiotic use may have decreased the number of organisms enough to produce a negative test, without having achieved true eradication.

Culture of gastric biopsy (choice A) is highly specific but requires both endoscopic biopsy and fastidious culture technique. Therefore, this method is not often used clinically for follow-up studies.

The rapid urease test (choice B) is performed on gastric tissue. It is rapid, specific, and sensitive, but requires endoscopy to obtain the biopsy fragment.

Qualitative assays of antibodies against H. pylori (choice C) may be positive for up to 3 years after eradication of the infection.

Quantitative assays of antibodies against H. pylori (choice D) drop slowly for up to a 3 years after eradication of the infection

How well did you know this?
1
Not at all
2
3
4
5
Perfectly
20
Q

A 52-year-old man comes to the physician because of slowly progressive weakness in his legs. He also complains of clumsiness with his right hand, which creates difficulties with buttons or turning keys. Examination reveals mild bilateral footdrop and leg weakness. Fasciculation and mild wasting are observed in the calf muscles. There is no spasticity or impaired sensation. The speech is normal, but fasciculation of the tongue is appreciated. Respiration, pulse, and temperature are normal. A muscle biopsy shows evidence of denervation with reinnervation. Which of the following is the most likely diagnosis?

(A) Amyotrophic lateral sclerosis (ALS)
(B) Charcot-Marie-Tooth disease
(C) Guillain-Barré syndrome
(D) Myasthenia gravis
(E) Spinal muscular atrophy

A

Respuesta: A

The correct answer is A. Flaccid paresis involving the lower extremities, footdrop, hand clumsiness, muscle wasting, and especially fasciculation in a middle-aged person are highly suggestive of amyotrophic lateral sclerosis (ALS). This results from degeneration of the motor neurons in the spinal cord (lower motor neuron) and leads to denervation of skeletal muscle. His tongue fasciculations result from degeneration of motor neurons of cranial nerve nuclei (XII). Continued bulbar involvement will likely eventually affect pharyngeal and facial musculature, leading to progressive dysarthia and dysphasia. Surviving neurons may reinnervate denervated myofibers by sprouting of their axons. The finding of denervation/reinnervation in a muscle biopsy is confirmatory of the clinical diagnosis. The patient will later develop evidence of corticospinal and corticobulbar (upper motor neuron) degeneration as his disease progresses.

Charcot-Marie-Tooth disease (choice B) is an autosomal recessive demyelinating disease of peripheral nerve that manifests in children or young adults with marked atrophy of the calf muscles and distal muscle weakness. For this reason, the disorder is also known as peroneal muscular atrophy.

Guillain-Barré syndrome (choice C) manifests with ascending paralysis (first the lower, then the upper extremities are involved) and results from a chronic inflammatory response leading to demyelination of peripheral nerves. It is often preceded by an upper respiratory tract infection.

Myasthenia gravis (choice D) is characterized by fluctuating muscle weakness that usually begins in the ocular muscles, resulting in diplopia and ptosis. Since the disorder is due to impaired cholinergic transmission at the neuromuscular junction, skeletal muscle biopsy is within normal limits at the light microscopic level.

Spinal muscular atrophy (SMA; choice E) is the infantile counterpart of ALS. SMA is a group of hereditary disorders, the most frequent form of which is Werdnig-Hoffmann disease (SMA type 1), which leads to death by the third year of life.

21
Q

An 18-year-old man is referred for evaluation of hypertension. On examination, he appears in no apparent discomfort and states that he has never had any health problems. His height is 175 cm (69 in), and his weight is 70 kg (154 lb). There is no pitting edema in the lower legs or jugular vein distention. The lungs are clear to auscultation. Blood pressure is 162/80 mm Hg in the upper extremities and 115/77 mm Hg in the lower extremities. Femoral pulses are weaker than radial pulses. A systolic murmur is appreciated at the base of the heart and is particularly intense in the back. The ECG shows changes consistent with left ventricular hypertrophy, and a chest x- ray film reveals notching of the inferior margins of the ribs. Which of the following is the most likely diagnosis?

(A) Atrial septal defect (ASD)
(B) Coarctation of the aorta
(C) Congenital aortic stenosis
(D) Congenital pulmonary stenosis
(E) Patent ductus arteriosus
(F) Tetralogy of Fallot
(G) Ventricular septal defect (VSD)

A

Respuesta B

The correct answer is B. The specific signs that suggest the correct diagnosis include the wide discrepancy between the blood pressure in the upper extremities and lower extremities, the systolic murmur heard on the back, and the notching of the ribs appreciated on x-ray. Coarctation of the aorta, in its most frequent (adult) type, consists of a stenotic aortic segment just distal to the origin of the left subclavian artery. Hypertension develops in the branches proximal to the stenosis, and hypotension in the aorta distal to it. In the most severe forms, the patients may develop left ventricular failure in infancy, but the most common presenting picture is that of a young adult with hypertension, which may lead to left ventricular hypertrophy or cerebral hemorrhage.

Atrial septal defect (ASD) (choice A) is generally asymptomatic. A large ASD usually leads to right ventricular failure in middle age. A systolic murmur is heard at the pulmonary area, and S2 is widely split.

Congenital aortic stenosis (choice C) gives rise to a harsh systolic murmur heard along the left sternal border and radiating to the neck. It is due to congenitally abnormal, usually bicuspid, aortic valves.

Congenital pulmonary stenosis (choice D) is a rare form of congenital valvular disease. Mild-to-moderate stenosis is usually asymptomatic, but severe cases result in right-sided heart failure or sudden death. A systolic murmur is heard at the second left intercostal space, often preceded by an ejection click.

Adults with a small or medium-size patent ductus arteriosus (choice E) are usually asymptomatic until middle age. This anomaly is associated with a characteristic continuous “machinery-like” murmur, which is maximal at the pulmonary area and often accompanied by a thrill.

Tetralogy of Fallot (choice F) is the most common form of cyanotic congenital heart disease. The four features include subpulmonary stenosis, ventricular septal defect, overriding aorta, and right ventricular hypertrophy. The degree of subpulmonary stenosis is the single most important determinant of the clinical severity and symptomatology. Most infants present with early cyanosis.

Ventricular septal defect (VSD) (choice G) is the most frequent congenital cardiac anomaly. Most cases are asymptomatic. Large VSDs lead to right ventricular overload and are associated with a harsh pansystolic murmur along the left sternal border associated with a thrill.

22
Q

A 50-year-old alcoholic man with chronic hepatitis C infection is brought to the emergency department by the police, who notice that the man has blisters and crusted lesions on sun-exposed skin of his face and lower arms. Plasma porphyrins are elevated, and follow-up studies demonstrate elevated uroporphyrin I in urine and isocoproporphyrin in feces. A biopsy of one of the lesions reveals subepidermal blisters with minimal inflammation, thickening of vessel walls in the papillary dermis, marked solar elastosis, and “caterpillar bodies” in the roof of the blister. Which of the following is the most likely diagnosis?
(A) Acute intermittent porphyria

(B) Delta-aminolevulinic acid dehydratase deficiency
(C) Erythropoietic protoporphyria
(D) Hereditary coproporphyria
(E) Porphyria cutanea tarda

A

Respuesta: E

The correct answer is E. Porphyria cutanea tarda is the most common of all of the porphyrias, and is consequently a likely target on the USMLE. It causes chronic blistering and crusting lesions on sun-exposed skin. The defective enzyme in heme synthesis is uroporphyrinogen decarboxylase. Precipitating factors include iron (even in normal amounts in some cases), estrogen use, alcohol use, and chronic hepatitis C infection. Skin biopsy can be helpful but is usually not completely specific (the “caterpillar bodies” in the question stem are clumps of basement membrane material). Porphyrin analyses demonstrate the findings in the question stem.

Acute intermittent porphyria (choice A) is one of the more common forms of porphyria and typically presents with severe abdominal pain.

Delta-aminolevulinic acid dehydratase deficiency (choice B) is a rare form of porphyria that can cause abdominal pain and hemolysis.

Erythropoietic porphyria (choice C) is one of the more common forms of porphyria and typically presents with acute, rather than chronic, photosensitivity with pain and swelling after sunlight exposure.

Hereditary coproporphyria (choice D) is a rare porphyria than presents with abdominal pain.

23
Q

A 40-year-old woman is admitted to the hospital because of fever, headache, confusion, and jaundice for 1 week. She underwent hysterectomy 2 months ago and began estrogen replacement therapy with ethinyl estradiol and a progestin. On admission, her temperature is 38.7 C (102 F), blood pressure is 140/90 mm Hg, pulse is 98/min, and respirations are 20/min. She appears disoriented to time and place. Physical examination reveals jaundiced sclerae and skin, purpura on the trunk, and bleeding gums. A guaiac stool test is positive for occult blood. Blood and urine cultures are
negative, show:

Hematocrit - 28%
Red blood cells - 2.5 million/mm3
Leukocytes - 10,000/mm3
Platelets - 15,000/mm3
Serum BUN - 40 mg/dL
Serum Creatinine - 2.8 mg/dL
Serum LDH - 800 U/L
Total Bilirubin - 4.0 mg/dL
Direct Bilirrubin - 0.8 mg/dL

Coagulation tests are within normal limits; fibrin-split products and Coombs test are negative. A peripheral blood smear shows schistocytes, helmet cells, and triangle cells. Which of the following is the most likely diagnosis?

(A) Disseminated intravascular coagulation (DIC)
(B) Evans syndrome
(C) Hemolytic-uremic syndrome (HUS)
(D) Idiopathic (autoimmune) thrombocytopenic purpura (ITP)
(E) Malignant hypertension
(F) Thrombotic thrombocytopenic purpura (TTP)

A

Respuesta F:

The correct answer is F. The key data to make a correct diagnosis include the following: severe thrombocytopenia, which results in a bleeding diathesis; elevated indirect bilirubin and high LDH with schistocytes in the blood smear, indicating microangiopathic hemolytic anemia; renal dysfunction (high creatinine); and neurologic and systemic symptoms (headache, confusion, and fever). Negative findings important to rule out similar conditions include a negative Coombs test and absence of fibrin split products. Thrombotic thrombocytopenic purpura (TTP) is a disorder of unclear pathogenesis, perhaps related to circulating platelet-agglutinating factors. It presents with a characteristic combination of microangiopathic hemolytic anemia, fever without infection, neurologic symptoms, bleeding diathesis secondary to thrombocytopenia, and renal impairment. This condition may be precipitated by pregnancy or use of estrogens.

Disseminated intravascular coagulation (DIC; choice A) can be differentiated from TTP because of abnormal coagulation tests. In DIC, microangiopathic hemolysis is also present, but prothrombin time (PT) is prolonged, fibrinogen levels are reduced, and fibrin split products are elevated.

Evans syndrome (choice B) refers to coexistence of autoimmune hemolytic anemia (positive Coombs test), and autoimmune thrombocytopenic purpura (see choice D).

Hemolytic-uremic syndrome (HUS; choice C) is not significantly different from TTP. The two conditions, in fact, are considered manifestations of the same pathogenetic spectrum. However, the vascular bed of the CNS is not involved in HUS; thus, mental status changes are not part of the clinical picture.

Idiopathic (autoimmune) thrombocytopenicpurpura (ITP; choice D) is an immune disorder caused by autoantibodies to platelet antigens. Systemic illness is not present in ITP, which is characterized by isolated thrombocytopenia without other hematologic abnormalities. Ten percent of cases will manifest in association with autoimmune hemolytic anemia (Evans syndrome).

Malignant hypertension (choice E) may cause microangiopathic hemolytic anemia. However, blood pressure values would be extremely elevated.

24
Q

A 30-year-old woman is seen in the emergency department because of severe abdominal pain. The pain has been progressively worse for the past day and is accompanied by nausea, vomiting, and diarrhea. The patient has had several similar episodes in the past that developed over the course of hours to days and lasted for a week or more. She has had surgeries for suspected appendicitis and suspected biliary tract disease, neither of which was confirmed once the abdomen was entered. Her temperature is 37 C (98.6 F), blood pressure is 140/100 mm Hg, pulse is 120/min, and respirations are 16/min. Abdominal examination demonstrates minimal abdominal tenderness and rebound tenderness. Measurement of which of the following will most likely confirm the diagnosis?

(A) Erythrocyte porphyrins
(B) Fecal porphyrins
(C) Plasma porphyrins
(D) Urine porphobilinogen
(E) Urine porphyrins

A

Respuesta: D

The correct answer is D. The porphyrias are due to metabolic defects in heme synthesis. Although they occur in a variety of forms, the most common are acute intermittent porphyria (which this patient has), erythropoietic protoporphyria (which presents with painful skin and acute swelling), and porphyria cutanea tarda (which presents with chronic blistering skin lesions). Acute intermittent porphyria characteristically presents with neurovisceral symptoms, which may mimic an acute abdomen. The abdominal pain produced is a nerve problem rather than an inflammation, which is why exploratory surgery in these patients is usually unrewarding. Patients with long-standing cases may have demonstrable damage to motor nerves as well, which typically begins as weakness in the shoulders and arms. The condition is relatively rare (although it is the most common acute porphyria); therefore, it is suspected more often than it is confirmed. The combination of complaints of severe pain, distraught behavior, and absence of physical findings may lead clinician to suspect the patient either is abusing drugs or has psychiatric problems. Failure to make the diagnosis also raises the risk of potentially dangerous complications because of drug interactions with the disease (barbiturates are a notorious offender). The biochemical defect in acute intermittent porphyria is a block in porphobilinogen deaminase. Determination of urinary porphobilinogen levels, which are best measured in a 24-hour urine collected during the period when the patient is symptomatic, is the most important screening test for acute intermittent porphyria. Aminolevulinic acid (ALA), which is an early precursor in heme synthesis, is also elevated in the urine. IV heme can be given for therapy.

Erythrocyte porphyrins (choice A) can be used to screen for erythropoietic protoporphyria.

Fecal porphyrins (choice B) are a second-line choice for screening for porphyria cutanea tarda.

Plasma porphyrins (choice C) can be used to screen for either porphyria cutanea tarda or erythropoietic porphyria.

Urine porphyrins (choice E) can be used to screen for porphyria cutanea tarda.

25
Q

An otherwise healthy 15-year-old boy undergoes evaluation for a newly diagnosed ventricular septal defect (VSD). The boy is at the 50th percentile for height and 45th percentile for weight. He plays soccer regularly with his school team and has never had any significant health problems. His blood pressure is 120/76 mm Hg, pulse is 72/min, and respirations are 11/min. An ECG is unremarkable, but echocardiography demonstrates a defect in the upper interventricular septum with cardiac chambers of normal size. Doppler ultrasound and radionuclide flow studies reveal a small left- to-right shunt with a pulmonary-to-systemic flow ratio of less than 1.5. Which of the following is the most significant complication of this patient’s condition?

(A) Arrhythmia
(B) Infective endocarditis
(C) Pulmonary hypertension
(D) Right-sided heart failure
(E) Shunt reversal

A

Respuesta: B

The correct answer is B. A small ventricular septal defect (VSD) is usually asymptomatic, manifesting with a systolic murmur sometimes associated with a thrill along the left sternal border. Patients with the typical murmur as the only manifestation have a normal life expectancy but are more prone to develop infective endocarditis. Thus, antibiotic prophylaxis is mandatory before dental procedures or other procedures that might produce bacteremia.

Arrhythmias (choice A) do not constitute a particular risk for patients with VSD.

A large VSD leads to a significant left-to-right shunt, which increases the right ventricular load and results in pulmonary hypertension (choice C) and right ventricular hypertrophy. The long-term effect of these hemodynamic alterations is right-sided heart failure (choice D). A shunt associated with a pulmonary-to- systemic flow ratio of less than 1.5 is hemodynamically inconsequential and should not be repaired surgically. Large shunts should be repaired to prevent late-onset pulmonary hypertension and heart failure.

Shunt reversal (choice E) develops when the right ventricular pressure exceeds that in the left ventricle and the shunt becomes right-to-left. This is a long-term complication of unrepaired large VSDs.

26
Q

A 33-year-old man presents with recurrent pain and swelling of the right knee. These symptoms started 4 years ago and are often precipitated by minor trauma. The joint fluid has been tested during acute episodes, and although clearly inflammatory, there has been no evidence of urate crystals, bacteria, or blood. Clinical examination reveals a moderately swollen and tender joint. A complete blood count (CBC), prothrombin time (PT), partial thromboplastin time (PTT), and serum chemistries are normal. X- ray films show speckling of the articular cartilage and increased joint space without marginal erosions. Which of the following is the most likely diagnosis?

(A) Hemophiliac arthritis
(B) Lyme arthritis
(C) Monarticular rheumatoid arthritis
(D) Osteoarthritis
(E) Pseudogout
(F) Psoriatic arthritis

A

Respuesta: E

The correct answer is E. Recurrent episodes of inflammatory arthritis, absence of urate crystals, and speckling (due to calcification) of the articular cartilage are virtually diagnostic of pseudogout. The knee is the most common joint involved. Identification of calcium pyrophosphate crystals in joint aspirates is diagnostic (weakly birefringent on polarized microscopy). It may be hereditary, may develop 24–28 hours after surgery, or may be associated with metabolic diseases, such as hyperparathyroidism, hemachromatosis, hypomagnesemia, acromegaly, Wilson disease, hypothyroidism, and gout.

Recurrent hemorrhages into joints, especially the knees, are characteristic of poorly treated hemophilia (choice A). Minor or unappreciated trauma may precipitate individual events. Healing is associated with inflammation and proliferation of the synovial membrane, and can lead to significant joint destruction. Widening of the intercondylar notch of the femur is characteristic. Other clinical features of hemophilia are invariably present.

Chronic monarticular or oligoarticular involvement, especially of the knee, is a feature of Lyme disease (choice B). The primary stage of the disease may be unrecognized. Calcification or erosions of the articular cartilage do not occur.

Rheumatoid arthritis (choice C) is uncommonly monarticular and enters the differential diagnosis. Rheumatoid factor in the joint fluid may be positive, even when the serum rheumatoid factor is not. The pattern of presentation is chronic, rather than recurrent acute, monarticular arthritis.

Osteoarthritis (choice D) at age 33, in the absence of prior major trauma, would be rare. Crystal arthropathy may coexist with degenerative arthritis, and the latter can progress more rapidly in the presence of crystal-induced damage. Acute exacerbation of stable osteoarthritis should be evaluated for coexistent crystal arthritis, as the management strategy may be altered.

Psoriatic arthritis (choice F) clasically involves distal interphalangeal joints and characteristic changes in the nails (pitting, transverse ridging, onycholysis). In the majority of cases, characteristic skin lesions are present before joint lesions appear. On x-ray, there are gross destructive changes in isolated small joints with associated erosions, ankylosis, and a “pencil-in-cup” appearance.

27
Q

A 25-year-old man with a 7-year history of ulcerative colitis consults a physician because of the insidious onset of progressive fatigue, pruritus, and jaundice. Laboratory studies are notable for elevation of serum alkaline phosphatase that is not accompanied by significant elevations of aspartate aminotransferase (AST) or alanine aminotransferase (ALT). An anti-mitochondrial antibody test is negative. Endoscopic retrograde cholangiography demonstrates multiple short strictures and saccular dilations of the biliary tree, both in extrahepatic and intrahepatic sites. Liver biopsy demonstrates bile duct proliferation, periductal fibrosis,
inflammation, and loss of bile ducts. Which of the following is the most likely explanation for these findings?

(A) Bile duct tumor
(B) Choledocholithiasis
(C) Congenital polycystic liver
(D) Primary biliary cirrhosis
(E) Primary sclerosing cholangitis

A

Respuesta: E

The correct answer is E. Primary sclerosing cholangitis is a condition in which fibrosing inflammation of the intrahepatic and extrahepatic bile duct system eventually lead to the obliteration of the bile ducts and development of cirrhosis. The underlying etiology of the damage is unclear, although toxic, infectious, and/or autoimmune mechanisms have been postulated. The clinical presentation illustrated in the question stem is typical. The association with inflammatory bowel disease, particularly ulcerative colitis, may provide a helpful clue. In some patients, AST and ALT may be mildly increased. The liver biopsy picture may be similar in primary sclerosing cholangitis and the related condition, primary biliary cirrhosis. The antimitochondrial antibody test can be helpful, because it is negative in primary sclerosing cholangitis and positive in roughly 95% of cases of primary biliary sclerosis. The most definitive study is endoscopic retrograde cholangiography, which establishes that the bile duct lesions extend outside the liver.

Although a bile duct tumor (choice A) can cause a localized dilatation of the bile duct system proximal to the lesion, it would not produce the characteristic pattern of alternating saccular dilations and strictures seen in this patient with endoscopic retrograde cholangiography.

Choledocholithiasis (choice B) is a stone in the extrahepatic bile duct system and would be seen on endoscopic retrograde cholangiography as a blockage to the flow of contrast dye.

Congenital polycystic liver (choice C) is a rare condition that can produce massive hepatomegaly but usually causes surprisingly few medical problems.

Primary biliary cirrhosis (choice D) can have a very similar biopsy appearance to primary sclerosing cholangitis. However, it does not have extrahepatic bile duct disease and usually is positive for antimitochondrial antibodies.

28
Q

One day after sustaining a laceration of the right hand at work, a 28-year-old man comes to the emergency department because of fever, chills, and painful swelling of the right arm. His temperature is 39.4 C (103 F), blood pressure is 110/65 mm Hg, pulse is 110/min, and respirations are 20/min. His right arm is swollen and extremely tender from the elbow up to the shoulder. The skin shows a diffuse dusky erythema. Sensation to touch and pain is reduced in the forearm and hand. A blood sample is immediately taken for cultures. Which of the following is the most appropriate next step in management?

(A) Supportive measures until culture results are available
(B) Treatment with clindamycin
(C) Treatment with penicillin V
(D) Parenteral treatment with penicillin G
(E) Parenteral treatment with vancomycin
(F) Surgical exploration and debridement

A

Respuesta: F

The correct answer is F. The acute symptomatology is consistent with necrotizing fasciitis, a severe infection of the subcutaneous tissue and fascia caused by group A streptococci. The bacteria gain entry into the subcutaneous tissue through a skin lesion and produce rapidly spreading cellulitis, combined with systemic signs and symptoms of toxemia. Anesthesia/hypoesthesia is a particularly important clue to the diagnosis. As soon as necrotizing fasciitis is suspected, surgical exploration and debridement is mandatory.

Limiting care to supportive measures until culture results are available (choice A) may permit rapid necrosis of the affected limb and death due to septic shock.

Treatment with clindamycin (choice B) or penicillin V (choice C) is appropriate for patients with streptococcal skin infections that are not sufficiently severe to warrant parenteral treatment.

Parenteral treatment with vancomycin (choice E) or penicillin G (choice D) is used for skin infections (especially erysipelas) due to streptococci. Penicillin remains the drug of choice for the treatment of streptococcal infections, but vancomycin may be used in severely penicillin-allergic patients. Erysipelas is not associated with such severe systemic signs of infection and usually involves the face.

29
Q

Over a 2-month period, a 50-year-old woman with a history of polycythemia vera develops abdominal pain and gross ascites. Physical examination demonstrates smooth hepatomegaly and mild jaundice. Pressure applied over the liver fails to distend the jugular veins. The abdominal wall is grossly edematous and shows a tortuous venous pattern. Edema of the legs is prominent. Which of the following is the most likely diagnosis?

(A) Budd-Chiari syndrome
(B) Hepatic cirrhosis
(C) Hepatocellular carcinoma
(D) Primary sclerosing cholangitis
(E) Steatosis

A

Respuesta: A

The correct answer is A. Budd-Chiari syndrome is a disorder in which hepatic venous outflow is obstructed because of thrombosis of the major hepatic veins. The blood clots may extend into the inferior vena cava, causing the abdominal wall signs and edema of the legs illustrated in the question stem. The condition is rare and typically occurs in the setting of a coagulopathy due to hematologic disease (myeloproliferative disorders, polycythemia vera, sickle cell disease, paroxysmal nocturnal hemoglobinuria) or in disorders of the coagulation (defects in normal inhibitors, such as antithrombin III, protein C, protein S, factor V Leiden; antiphospholipid antibodies; and possibly high estrogen states, such as oral contraceptive use or pregnancy). The disorder either presents with acute hepatic failure, or, more commonly, progresses over several months. Early recognition of the syndrome is important so that thrombolytics and long-term anticoagulation can be given. Some patients respond to medical management, whereas others with fulminant or end-stage disease may require liver transplantation.

Hepatic cirrhosis (choice B) develops slowly and produces a nodular liver.

Hepatocellular carcinoma (choice C) would produce a liver mass. Ascites is a late finding and usually develops slowly.

Primary sclerosing cholangitis (choice D) is an inflammation of the bile ducts that does not usually produce ascites unless it has progressed to cirrhosis.

Steatosis (choice E), or fatty liver, does not produce ascites.

30
Q

A previously healthy, 48-year-old woman presents with easy fatigability, anorexia, and a 5-kg (11-lb) weight loss for 2 months. She also reports night sweats and occasional temperatures to 38 C (100 F). On examination, the spleen is palpable 4 cm below the left costal arch. Blood tests reveal a hemoglobin of 16 g/dL, 500,000 platelets/mm3, and 170,000 leukocytes/mm3. The differential count shows a left shift, with predominance of mature granulocytes, bands, and metamyelocytes; blasts are 3%. Serum chemistry is remarkable for low leukocyte alkaline phosphatase and high uric acid. Cytogenetic studies demonstrate the presence of the Philadelphia chromosome in white blood cells. Which of the following is the most likely diagnosis?

(A) Acute myelogenous leukemia (AML)
(B) Chronic lymphocytic leukemia (CLL)
(C) Chronic myelogenous leukemia (CML)
(B) Leukemoid reaction
(E) Myelofibrosis

A

Respuesta: C

The correct answer is C. Chronic myelogenous leukemia (CML) is a myeloproliferative disorder developing from neoplastic transformation of a bone marrow stem cell that still retains the capacity to differentiate along erythrocytic, megakaryocytic, granulocytic, or monocytic lines. Thus, the peripheral blood in CML is characterized by striking leukocytosis, with myeloid cells present at different degrees of differentiation and in direct proportion to their degree of maturation. Therefore, immature cells—blasts and promyelocytes—are less numerous than mature granulocytes or monocytes. Blasts are usually less than 5%. CML is characterized by the presence of the Philadelphia chromosome, arising from a balanced translocation involving 9q and 22q. This results in the formation of a bcr/abl fusion gene encoding a protein with tyrosine kinase activity. The presence of the Philadelphia chromosome is definitive evidence for CML.

Acute myelogenous leukemia (AML; choice A) results from neoplastic transformation of a stem cell that has lost the capacity to differentiate fully into mature blood cells. Thus, large numbers of blasts are present in peripheral blood and bone marrow. The Philadelphia chromosome is absent in most cases. The morphology of leukemic cells and their cytogenetic abnormalities define the seven subtypes of AML. The most frequent form (with full myeloid maturation), is associated with t(8;21).

Chronic lymphocytic leukemia (CLL; choice B) and its lymphomatous counterpart—small lymphocytic lymphoma—derive from neoplastic proliferation of small, well-differentiated lymphocytes. CLL is associated with marked lymphocytosis (up to 200,000/mm3) in peripheral blood. Patients present with fatigue and lymphadenopathy, but often lymphocytosis is discovered incidentally.

A leukemoid reaction (choice D) is an exuberant form of leukocytosis (with leukocyte counts up to 50,000/mm3) that may follow infections. Sometimes it is difficult to distinguish between true leukemia and a leukemoid reaction, but presence of the Philadelphia chromosome rules out the latter. Leukocyte alkaline phosphatase is elevated in leukemoid reaction, low in CML.

Myelofibrosis (choice E) is a chronic myeloproliferative disorder characterized by marrow fibrosis and widespread extramedullary hematopoiesis, resulting in massive splenomegaly. The Philadelphia chromosome is absent. Teardrop erythrocytes are characteristically present in peripheral blood smears.

31
Q

A 60-year-old alcoholic man is admitted to the emergency department with hematemesis. His pulse is 110/min, blood pressure is 100/60 mm Hg, and respirations are 19/min. He has multiple spider angiomata on his back and chest, with bilateral gynecomastia. Abdominal examination is significant for hepatosplenomegaly, and his abdomen is distended and tympanic on percussion; a fluid level is easily detectable. His testicles are small, and a rectal exam-ination produces guaiac-negative stool. His hematocrit is 23%. After placement of a nasogastric tube, 400 mL of bright red blood is evacuated. After initial fluid resuscitation, which of the following is the most appropriate next step in management?

(A) Barium swallow
(B) Esophageal balloon tamponade
(C) Esophagogastroscopy
(D) Exploratory celiotomy
(E) Selective angiography
(F) Transjugular intrahepatic portosystemic shunt

A

Respuesta: C

The correct answer is C. The patient has a history of alcohol abuse and signs of chronic liver disease, and now presents with an upper gastrointestinal bleed (UGIB). The sudden onset of hematemesis in the absence of abdominal pain in a patient with chronic liver disease is consistent with hemorrhage from esophageal varices. However, one half to two thirds of patients with cirrhosis who present with a UGIB have a nonvariceal source, and many have more than one source. Therefore, prompt identification of the origin of bleeding is crucial to guiding therapy. Esophagogastroscopy is the appropriate first step in identifying, and in many cases treating, the source of bleeding.

Barium swallow (choice A) has no role in the diagnosis of a UGIB.

Esophageal balloon tamponade (choice B) is used in patients with a confirmed diagnosis of variceal hemorrhage who continue to bleed despite endoscopic treatment.

Emergent celiotomy (choice D) is reserved for patients who continue to bleed despite endoscopic therapy.

Angiography (choice E) is used only when esophagogastroscopy has failed to reveal a bleeding source.

A transjugular intrahepatic portosystemic shunt (TIPS) (choice F) is a percutaneous connection within the liver, between the portal and systemic circulations. TIPS placement diverts portal blood flow into the hepatic vein and thus decreases the pressure gradient in patients with portal venous hypertension. TIPS is indicated in acute variceal bleeding that cannot be successfully controlled with medical treatment. Therefore, it would not be used in this patient until other measures have been attempted.

32
Q

A 72-year-old man with a history of peripheral vascular disease and recurrent chest pain underwent cardiac catheterization 3 hours ago. Angiography showed 80% occlusion of the left main coronary artery. He now complains of diarrhea and severe constant mid- abdominal pain. On examination, his temperature is 37.2 C (99 F), blood pressure is 170/90 mm Hg, pulse is 102/min, and respirations are 22/min. The lungs are clear, and the abdomen is soft and nondistended without focal tenderness. Bowel sounds are hypoactive, and no masses are palpable. Rectal examination reveals occult blood in the stool. Which of the following is the most likely diagnosis?

(A) Gastric ulcer
(B) Mesenteric ischemia
(C) Pancreatitis
(D) Perforated duodenal ulcer
(E) Staphylococcal gastroenteritis

A

Respuesta: B

The correct answer is B. This patient has mesenteric arterial occlusion with ischemia as a complication of an angiographic procedure. This is a typical case of iatrogenic occlusion. This patient is very susceptible to this complication because of his history of peripheral vascular disease, coronary artery disease, and severe atherosclerotic disease. Iatrogenic mesenteric ischemia occurs most commonly after angiographic procedures or operations on the aorta. Angiography may cause intestinal ischemia by dislodging of atheromata from a diseased vessel wall, by dissection of the vessel, or by formation of the intimal flap. Mesenteric ischemia is accompanied by sudden severe epigastric and mid-abdominal pain. Forceful vomiting and evacuation of stool commonly follow the onset of pain. Early after embolization, physical examination of the abdomen may be entirely unremarkable. Later, a classic presentation is severe abdominal pain out of proportion to physical findings. Abdominal distention, guarding, and absence of bowel sounds are associated with intestinal infarction and imply disease progression. Stool may be positive for occult blood. No laboratory tests are pathognomonic for mesenteric embolism or visceral ischemia.

Pain from a gastric ulcer (choice A) would not occur suddenly. In addition, if this patient had a perforated gastric ulcer, he would have some local signs of peritonitis, and the abdominal examination would correlate more closely with the degree of abdominal pain, unlike the situation with mesenteric ischemia.

Pancreatitis (choice C) does not occur as a result of cardiac catheterization. It can occur after endoscopic retrograde cholangiopancreatography (ERCP) with dye injection into the pancreatic duct. In addition, pancreatitis does not lead to occult blood in the stool.

Abdominal examination of a patient with a perforated duodenal ulcer (choice D) would reveal some local signs of peritonitis, and the patient’s symptoms would not be so “out of proportion” to the abdominal examination.

Staphylococcal food poisoning/gastroenteritis (choice E) usually occurs 3-6 hours after ingestion of contaminated food. However, this patient had a cardiac catheterization; he did not eat a tuna sandwich with contaminated mayonnaise. Furthermore, vomiting is usually the prominent symptom with staphylococcal food poisoning.

33
Q

A 22-year-old man presents with a 6-month history of non-bloody diarrhea, malaise, recurrent abdominal cramps, and temperatures to 38.5 C (101.3). At this time, he is afebrile. Examination reveals a palpable, ill-defined mass in the right lower quadrant of the abdomen. Palpation causes local tenderness without guarding. Oral ulcers are also noted. Laboratory studies show:

Hemoglobin: 11.5 g/dL
Leukocyte count: 12,800/mm3
Albumin: 2.8 g/dL
Sedimentation rate: 12.45 mm/h

An upper gastrointestinal series with small bowel follow-through reveals a sharply demarcated stenotic segment in the terminal ileum. The patient undergoes laparotomy, and the involved segment of ileum is resected. Which of the following is the most likely diagnosis?

(A) Carcinoma
(B) Celiac disease
(C) Chronic appendicitis
(D) Crohn disease
(E) Pseudomembranous colitis
(F) Ulcerative colitis

A

Respuesta: D

The correct answer is D. The clinical picture is consistent with Crohn disease (CD). Nonbloody diarrhea, abdominal pain and cramps, malaise, and low-grade fever are the most common, but rather nonspecific, presenting symptoms. CD affects the terminal ileum most frequently (hence the old designation of terminal ileitis), so that tenderness and a mass can often be detected on palpation in the lower left quadrant of the abdomen. The most characteristic signs of CD include sharp demarcation of affected segments from adjacent non-involved loops and presence of non-necrotizing granulomas in biopsies. Strictures resulting in bowel obstruction may necessitate surgical resection, as in this case.
Carcinoma (choice A) is highly unlikely, considering the clinical picture, location of lesion (small bowel cancer is rare), and young age of the patient.
Celiac disease (choice B) is a chronic diarrheal disease that is characterized by intestinal malabsorption and precipitated by indigestion of gluten-containing foods. The disease also presents with nonbloody diarrhea, cramps, and abdominal distension due to fluid-and gas-filled intestinal loops. Distinguishing features on small bowel series are flocculation of barium, small bowel dilatation, and flattening of normal mucosal fold pattern. Occasionally, ulceration and strictures may occur.
Chronic appendicitis (choice C) is a rather controversial entity. Repeated bouts of acute appendicitis, especially when incompletely controlled with antibiotic therapy, may rarely result in periappendiceal and pericolic adhesions.
Pseudomembranous colitis (choice E) is due to the toxins produced by Clostridium difficile. This condition develops as a complication of broad-spectrum antibiotic treatment, particularly in hospitalized patients. It affects the colon (not the ileum) and manifests with greenish, foul-smelling diarrhea. Endoscopic and pathologic examination reveal the characteristic yellow-green plaques adherent to the mucosa.
Ulcerative colitis (UC) (choice F) shares with CD many clinical and pathologic features, so that an “umbrella” designation of inflammatory bowel disease is used to refer to both conditions when a specific diagnosis is not yet made. Features not consistent with UC (thus favoring CD) include involvement of ileum (which is exceptional in UC), sharp demarcation of the affected bowel segment, and presence of granulomas on histologic examination.

34
Q

A 55-year-old male smoker with diabetes and hypertension presents with complaints of chest pain on exertion. Exercise stress testing shows reversible ischemia in the anteroseptal portion of the heart after exercising 4 minutes. Cardiac catheterization reveals an 80% stenosis of the left main coronary artery. Which of the following is the most appropriate intervention?
(A) Re-examination in 6 months or sooner if symptoms worsen (B) Beta blocker
(C) Sublingual nitroglycerin as needed
(D) Percutaneous balloon angioplasty
(E) Coronary artery bypass grafting

A

Respuesta: E

The correct answer is E. This patient has exertional angina, an abnormal stress test, and occlusive atherosclerotic disease of the left main coronary artery. He is at high risk for adverse cardiac events, including myocardial infarction and death, and should receive treatment. The treatment of choice is coronary artery bypass grafting, which has been shown to decrease symptoms and mortality in patients with left main coronary artery disease.
The patient needs immediate treatment, so instructing the patient to return in 6 months (choice A) is unwise.
Beta blockers (choice B) decrease the anginal symptoms of and mortality from coronary artery disease, but are not a replacement for definitive revascularization procedures. Furthermore, their use may be contraindicated in diabetics, as these agents can intensify hypoglycemia while masking hypoglycemic symptoms.
Sublingual nitroglycerin (choice C) is helpful in managing the anginal pain of coronary artery disease but does nothing to alter the course of the disease or decrease mortality.
Angioplasty (choice D) is not an option in the therapy of left main disease, since inflating the balloon completely occludes the lumen of the artery and transiently interrupts all blood flow to the myocardium.

35
Q

A 55-year-old woman with a history of rheumatoid arthritis since age 28 presents with new symptoms of recent onset. She complains of persistent fatigue and weight loss, diarrhea, and leg swelling. Furthermore, she has had pain in her wrists with a tingling sensation at the tips of thumb and first two digits, which bothers her especially at night. Examination reveals waxy skin plaques about the axillary folds, macroglossia, hepatosplenomegaly, pitting edema of the legs, and peripheral neuropathy. The stool guaiac test is positive. Serum chemistry studies show only mild hypoalbuminemia. Proteinuria in the nephrotic range is found on urinalysis, without hematuria. Which of the following is the most appropriate next step in diagnosis?

(A) Electrophoresis of serum proteins
(B) X-rays of vertebral column and skull
(C) Biopsy of skin, rectal mucosa, or abdominal fat
(D) Renal biopsy
(E) Endomyocardial biopsy

A

Respuesta: C

The correct answer is C. Long-standing rheumatoid arthritis is currently one of the most common causes of systemic amyloidosis. This may give rise to a complex clinical picture resulting from amyloid deposition in the skin, kidneys, tongue, gastrointestinal tract, or peripheral nerves, for example. Carpal tunnel syndrome, skin plaques in the axillary region, nephrotic syndrome, hepatosplenomegaly, and macroglossia, are among the most common manifestations. Chronic diarrhea with malabsorption and occult bleeding are frequent as well. Biopsies of the skin, rectal mucosa, abdominal fat pad, or gingiva are the most helpful to confirm the clinical diagnosis.

Electrophoresis of serum proteins (choice A) is useful in demonstrating monoclonal gammopathy, which is usually associated with plasma cell neoplasia or dyscrasia. Multiple myeloma is a frequent cause of amyloidosis, but this condition is accompanied by osteolytic bone lesions, bone pains, spontaneous fractures, anemia, and propensity for infections. X-rays of vertebral column and skull (choice B) should be considered when there is clinical evidence suggesting multiple myeloma as the underlying cause of amyloidosis. The clinical history in this case suggests rheumatoid arthritis as the most likely underlying etiology.

Renal biopsy (choice D) and endomyocardial biopsy (choice E) may also be used to demonstrate amyloid deposition in the myocardium or kidney, but these should be used only when other, less invasive procedures have been ineffective.

36
Q

A 68-year-old woman complains of numbness and difficulty walking. Family members mention that her behavior has also become erratic over the past several months. She has a past medical history of Crohn disease, for which she underwent ileal resection 10 years ago. Laboratory results indicate a hematocrit of 20% and a mean corpuscular volume (MCV) of 110 μm3. Blood smear shows large red cells with hypersegmented neutrophils. Which of the following is the most likely cause of these findings?

(A) Ferrochelatase deficiency
(B) Folate deficiency
(C) Hydroxymethylbilane synthase deficiency
(D) Intrinsic factor deficiency
(E) Iron deficiency anemia
(F) Vitamin malabsorption

A

Respuesta: F

The correct answer is F. The ileal resection indicates that this patient is not absorbing the vitamin B12-intrinsic factor complex, leading to vitamin B12 deficiency. This deficiency can result in dorsal column degeneration, causing the observed neurologic symptoms. Vitamin B12 deficiency will also cause macrocytic anemia and lead to the development of hypersegmented neutrophils.

Ferrochelatase deficiency (choice A) would lead to erythropoietic protoporphyria, which is inherited as an autosomal dominant trait. Skin photosensitivity begins in childhood. The CNS is spared. It is similar to sideroblastic anemia in its hematologic manifestation.

Folate deficiency (choice B) should not produce the neurologic symptoms observed, although it would certainly cause macrocytic anemia.

Hydroxymethylbilane synthase (HMB-synthase) deficiency (choice C) causes acute intermittent porphyria. It is inherited in an autosomal dominant manner. Abdominal pain is the most common symptom. Acute attacks may be manifested by anxiety, insomnia, depression, hallucinations, and paranoia.

Intrinsic factor deficiency (choice D) is one of the causes of vitamin B12 deficiency. However, it occurs in postgastrectomy patients or in patients with pernicious anemia. The history of ileal resection points to malabsorption of the vitamin B12-intrinsic factor complex from the terminal small bowel as the cause of vitamin B12 deficiency.

Iron deficiency anemia (choice E) could result from Crohn disease secondary to the chronic bloody diarrhea. However, this would result in a microcytic anemia.

37
Q

A 60-year-old man consults a physician because of intense forefoot pain on one side that is triggered with exercise. One week earlier, he ran a 25-km marathon, and the pain started midway through the race. He continued to run anyway, as he wished to finish the race, and the pain disappeared within seconds of finishing. He rested the next day, and then began to run the following day. The pain started much more quickly this time, and he stopped running when it did. The pain persisted for a few minutes and then stopped. He then rested for 3 days, to give his foot time to heal. When he began to run again, his forefoot began to hurt almost immediately, and he decided to consult a physician. Which of the following is the most likely diagnosis?

(A) Achilles tendinitis
(B) Epiphysitis of the calcaneus
(C) Fracture of the posterolateral talar tubercle
(D) Metatarsal stress fracture
(E) Posterior Achilles tendon bursitis

A

Respuesta: D

The correct answer is D. This history is typical for metatarsal stress fracture. This common sports injury usually involves the second, third, or fourth metatarsals, which have thin diaphyses. Diagnosis is usually by history; radiography may not demonstrate the fracture until the 2nd or 3rd week after injury, when a callus forms. Palpating the swollen area of the foot causes pain. Risk factors include a cavus (high arched) foot, osteoporosis, and shoes with inadequate shock-absorption. This type of fracture rarely requires a cast, and healing typically takes 3-12 weeks.

Achilles tendinitis (choice A), epiphysitis of the calcaneus (choice B), and posterior Achilles tendon bursitis (choice E) cause heel pain.

Fracture of the posterolateral talar tubercle (choice C) causes pain behind the ankle.

38
Q

A 44-year-old man is admitted for treatment of infection with Staphylococcus aureus. His course is complicated by the development of dry gangrene on his right toe. His blood pressure drops to 85/48 mm Hg, and he is bleeding from multiple sites. A peripheral blood smear shows multiple schistocytes. Laboratory evaluation of prothrombin time (PT), partial thromboplastin time (PTT), and platelets would most likely show which of the following?

(A) Elevated PT, elevated PTT, decreased platelets
(B) Elevated PT, elevated PTT, elevated platelets
(C) Elevated PT, normal PTT, decreased platelets
(D) Normal PT, elevated PTT, decreased platelets
(E) Normal PT, normal PTT, decreased platelets

A

Respuesta: A

The correct answer is A. The patient has disseminated intravascular coagulation (DIC), which can be caused by trauma, shock, malignancy, or obstetric complications. It involves massive activation of coagulation, overwhelming of inhibitors, and depletion of factors. This results in platelet consumption, elevation of PT and PTT, and appearance of schistocytes on peripheral blood smears.

Choice B may be seen in a patient with coagulation factor deficiency and an acute infection. Platelets are an acute phase reactant that may rise in any condition in which the body is stressed.

Choice C may be seen in a patient on Coumadin, which inhibits the function of the extrinsic coagulation pathway.

Choice D may be noted in a patient with thrombocytopenia who is on an anticoagulant, such as heparin. Factor deficiency in the intrinsic pathway may cause a similar increase in PTT.

Choice E may be seen in isolated thrombocytopenia, rather than DIC.

39
Q

A 75-year-old African American man is transferred from a nursing home to the emergency department. For the past 2 hours, the man has had acute, left-sided abdominal pain that started in the left iliac fossa. While waiting to be seen, he suddenly passes stool mixed with dark blood clots. His temperature is 38.1 C (100.6 F), blood pressure is 110/85 mm Hg, pulse is 120/min, and respirations are 18/min. Abdominal examination demonstrates localized tenderness along the descending colon. An x-ray film taken after barium enema excludes intraabdominal free air. The large bowel lumen is decreased and irregular, with mucosal thickening, and there is gas in the wall of the colon. Which of the following is the most likely diagnosis?

(A) Appendicitis
(B) Colon cancer
(C) Crohn disease
(D) Ischemic colitis
(E) Ulcerative colitis

A

Respuesta: D

The correct answer is D. This is the classic presentation of ischemic colitis. Patients are typically in the 6th to 8th decade of life, and can be of any race. In this condition, there is an inflammation of the colon resulting from ischemic damage to the colon wall. Classic x-ray findings are mucosal edema with associated hematoma formation. This is often referred to as thumbprinting. Thickened edematous tissues encroach on air or contrast fluid lumen. Gas in the wall of the colon (pneumatosis coli) is highly suggestive of ischemic colitis. There are many possible causes of ischemic damage, including occlusion of a major artery or vein, small vessel disease, severe hypotension, and intestinal obstruction. Severe ischemic colitis usually requires surgical resection of the involved bowel segment.

Appendicitis (choice A) involves the right lower quadrant of the abdomen.

Colon cancer (choice B) would produce an ulcer or a mass visible on barium enema.

Newly diagnosed Crohn disease (choice C) or ulcerative colitis (choice E) would be unusual in a 75-year-old, and would produce larger areas of mucosal ulceration and irregularity on barium enema.

40
Q

A 37-year-old woman comes to medical attention with a 3-month history of recurrent episodes of hemoptysis and low-grade fever. Her temperature is 37.6 C (100 F), blood pressure is 123/82 mm Hg, pulse is 74/min, and respirations are 14/min. A chest x-ray film demonstrates bilateral perihilar infiltrates. In addition to mild anemia and leukocytosis, laboratory analysis shows:

Blood serum BUN: 23 mg/dL
Creatinine: 2.2 mg/dL
Erythrocyte sedimentation rate: 50/min
Complement levels: Normal
Antinuclear antibodies: Negative
Antineutrophil cytoplasmic antibodies (ANCA): Positive, c-ANCA pattern

Urinalysis:
- Protein: 2+
- Red blood cells: 10/hpf

A renal biopsy shows granulomas with necrotizing vasculitis and scattered mesangial deposits of immunoglobulin and complement. Which of the following is the most likely diagnosis?
(A) Churg-Strauss syndrome
(B) Goodpasture syndrome
(C) Polyarteritis nodosa
(D) Tuberculosis
(E) Wegener granulomatosis

A

Respuesta: E

The correct answer is E. Wegener granulomatosis develops over a period of months, usually presenting with upper or lower respiratory symptoms, such as sinusitis, otitis, and hemoptysis. Renal manifestations are due to the same pathologic process, namely a necrotizing granulomatous vasculitis. IgG and complement deposits are found in renal biopsies, but the most specific laboratory finding is the presence of circulating c-ANCA. This form of glomerulonephritis is classified among the pauci- immune glomerulonephritides, which also include Churg-Strauss syndrome (choice A) and polyarteritis nodosa (choice C). The latter two, however, are usually associated with circulating p-ANCA. Asthma and blood eosinophilia are characteristic of Churg-Strauss syndrome, whereas palpable purpura is associated with polyarteritis nodosa. The cytoplasmic pattern of ANCA (c-ANCA) is due to antibodies directed to the neutrophil proteinase-3, and the perinuclear pattern of ANCA (p-ANCA) is due to antibodies against myeloperoxidase.

Goodpasture syndrome (choice B) is not associated with circulating ANCA. It manifests with hemoptysis and severe glomerulonephritis, often evolving to acute renal failure. Its pathogenesis is related to antibodies against the globular domain of type IV collagen, depositing in a linear pattern along the glomerular and pulmonary basement membranes. Anti-glomerular basement membrane antibodies are also detected in the serum.

Tuberculosis (choice D) may affect the kidneys, but it does not produce a picture of necrotizing vasculitis and it is not associated with ANCA.

41
Q

A 65-year-old African American man presents with dull, persistent abdominal pain with radiation to the back. He has lost 20 lb over the past 3 months. His appetite is markedly decreased, with associated nausea and vomiting. Laboratory analysis reveals a blood glucose of 280 mg/dL. Physical examination is remarkable for midepigastric tenderness and a positive Homans sign in the left calf. He has no significant past medical history. Which of the following is the most likely diagnosis?

(A) Chronic pancreatitis
(B) Gastric cancer
(C) Hepatic cancer
(D) Pancreatic cancer
(E) Type 2 diabetes mellitus

A

Respuesta: D

The correct answer is D. Pancreatic cancer typically has a subtle presentation. This case has many of the classic signs of pancreatic cancer, such as dull, persistent abdominal pain. This pain differs from the burning, episodic pain associated with ulcer disease. Weight loss is often a sign of malignancy, and glucose intolerance is suggestive of destruction of the beta islet cells, which produce insulin. Pancreatic cancer often causes a hypercoagulable state, which can lead to deep venous thrombosis, evidenced in this patient by Homans sign (increased resistance or pain on dorsiflexion of the foot). All this together strongly suggests pancreatic cancer.

Chronic pancreatitis (choice A) presents with atypical abdominal pain radiating to the back, which is usually persistent and not relieved by antacids. Weight loss and signs of malabsorption, such as abnormal stool, are common. Alcoholism is the most common cause in adults, whereas cystic fibrosis is the most common cause in children.

Patients with gastric cancer (choice B) usually present with chronic, noncolicky epigastric pain that ranges from postprandial fullness to severe, steady pain. There is associated anorexia, weight loss, and anemia from blood loss. Late signs include an enlarged liver, Virchow node (supraclavicular), and Sister Mary Joseph nodule. Glucose intolerance is not a feature of gastric cancer.

Hepatic cancer (choice C) presents with right upper quadrant pain. A mass can often be palpated in the liver. Increased alpha- fetoprotein and alkaline phosphatase are common laboratory features. Patients usually have a history of chronic liver disease.

Type 2 diabetes mellitus (choice E) typically presents with polyphagia, polydipsia, and polyuria. Each of these symptoms is caused by elevated serum glucose. Type 1 diabetes is characterized by an acute onset over several days and may be associated with weight loss or may even present as diabetic ketoacidosis. Type 2 diabetes usually has a more gradual onset, classically occurring in obese patients who are often asymptomatic.

42
Q

A 56-year-old man with a 5-year history of hypertension treated with diuretics and enalapril comes to medical attention because of right flank pain. His temperature is 37 C (98.6 F) and blood pressure is 145/95 mm Hg. Physical examination shows tenderness in the right costovertebral angle and bilaterally enlarged kidneys. A urine dipstick test reveals microhematuria. Which of the following is the most appropriate next step in diagnosis?

(A) Cytologic examination of urine
(B) Ultrasonography
(C) CT scan of the abdomen
(D) Intravenous pyelography (IVP)
(E) Renal biopsy

A

Respuesta: B

The correct answer is B. The combination of hypertension and bilaterally enlarged kidneys is highly suggestive of autosomal dominant (adult) polycystic kidney disease, which is often associated with microhematuria as well. A positive family history may be present. Ultrasonography is the diagnostic procedure of choice, since it is extremely sensitive for detecting cystic formations within the kidneys. The disease frequently manifests in young adult or middle-age life. Hypertension is frequently the presenting sign.

Cytologic examination of urine (choice A) would show no changes (besides hematuria) in this case. It is usually negative in patients with renal neoplasms as well.

CT scan of the abdomen (choice C) is also a sensitive diagnostic tool in the study of renal masses, but should follow ultrasonography in this setting.

Intravenous pyelography (IVP; choice D) results in good visualization of the kidneys and urinary tract and is a functional test as well. However, it requires contrast administration and does not discriminate between solid and cystic masses.

Renal biopsy (choice E) is not indicated for the study of polycystic renal disease. Fine needle aspiration may be used to analyze the content of an isolated cystic mass in case imaging studies have failed to determine whether it is of benign or malignant nature.

43
Q

A 54-year-old man comes to the physician because of right leg pain for 4 months. He describes it as a deep pain in the calf muscles that occurs intermittently after walking for a given distance and subsides after a few minutes of rest. He has been smoking 20-30 cigarettes a day for the past 25 years and drinks alcohol occasionally. His temperature is 36.8 C (98.2 F), blood pressure is 154/93 mm Hg, pulse is 74/min, and respirations are 13/min. Examination shows thinning of the right calf compared with the left, as well as hair loss on the right leg. Reflexes and sensation are normal. Femoral pulses are normal on both sides, but the right popliteal and pedal pulses are barely detectable. Which of the following is the most likely diagnosis?

(A) Atherosclerotic stenosis of the common iliac artery
(B) Atherosclerotic stenosis of the distal superficial femoral artery
(C) Peripheral polyneuropathy
(D) Prolapse of intervertebral disk in the lumbar spine
(E) Thromboangiitis obliterans

A

Respuesta B

The correct answer is B. The clinical manifestations are characteristic of intermittent claudication, characterized by intermittent ischemic pain arising from inadequacy of blood flow secondary to arterial stenosis. The pain manifests when muscle oxygen demands increase (usually during walking) and subsides at rest. The most common cause of this picture is atherosclerosis of the arteries to the lower extremities, affecting the external iliac or the superficial femoral/popliteal segments. In this case, the presence of normal femoral pulses and the weakening of popliteal and pedal pulses on the affected site point to the site of stenosis. The distal segment of the superficial femoral artery is often the first to be affected by atherosclerotic change, which then progresses to involve the popliteal artery. Several risk factors for atherosclerosis (smoking, hypertension) are present in this patient.

Atherosclerotic stenosis of the common iliac artery (choice A) would give rise to similar clinical manifestations, but the femoral pulse would be weakened or absent.

The pain caused by peripheral polyneuropathy (choice C) usually affects the distal extremities in a symmetric fashion, manifests at rest and often at night, and is frequently associated with paresthesia/hypoesthesia and decreased reflexes. Diabetes and alcohol abuse are probably the most common causes in Western countries.

Prolapse of an intervertebral disk in the lumbar spine (choice D) produces manifestations due to compression of one of the spinal roots, most commonly L5 or S1. The patient experiences lumbar pain, as well as pain and paresthesias in a radicular distribution. Decreased sensation and reduced reflexes may develop in long- standing cases.

Thromboangiitis obliterans (choice E) usually occurs in young males who are heavy smokers. The disease results from acute inflammation of the whole neurovascular bundles, including major arteries, veins, and nerves. Thus, the symptomatology is due to obstruction of arterial and venous blood flow, as well as nerve involvement. Intermittent claudication is common but is associated with signs of venous thrombosis and pain at rest. The disorder often leads to amputation of fingers and toes.

44
Q

A 60-year-old woman presents with a gradual increase in abdominal girth for the past 2 months. She denies any history of alcohol use and had a positive PPD test many years ago. Physical examination reveals ascites and dullness to percussion at her right lung base. She has no spider angiomata, caput medusae, palmar erythema, or lower extremity edema. A chest x-ray film shows a right pleural effusion. Laboratory studies reveal the following:

  • Aspartate aminotransferase (AST): 12 U/L
  • Alanine aminotransferase (ALT): 11 U/L
  • Alkaline phosphatase: 80 U/L
  • Total bilirrubin: 0.5 mg/dL
  • Direct bilirrubin: 0.1 mg/dL
  • HBsAg: Negative
  • HBsAb: Positive
  • HBcAb: Positive
  • HCAb: Negative

An abdominal paracentesis reveals a transudative fluid with 50 white blood cells and few mononuclear cells. Cytology of the fluid is negative for malignant cells and acid-fast bacilli. Which of the following is the most likely diagnosis?

(A) Alcoholic cirrhosis
(B) Hepatitis B cirrhosis
(C) Hepatitis C cirrhosis
(D) Meigs syndrome
(E) Peritoneal tuberculosis

A

Respuesta: D

The correct answer is D. Meigs syndrome usually consists of a triad of benign fibroma or other ovarian tumors, ascites, and large effusions (usually on the right side). The symptoms, which are most commonly seen shortly after menopause, consist of a chronic illness, chest pain, and increased abdominal girth. Fluid moves from the abdomen to the thorax through small diaphragmatic defects or via lymphatics. When the condition is suspected, an abdominal CT scan and a pelvic examination should be performed. The removal of the ovarian tumor results in resolution of the effusion within 2-3 weeks.

Alcoholic cirrhosis (choice A) is unlikely, not because the patient denies alcohol use, but because she has normal liver function tests.

The presence of hepatitis B surface antibody and the absence of hepatitis B surface antigen imply immunity to hepatitis B, so she cannot have this type of cirrhosis (choice B).

It is unlikely that the patient would have hepatitis C cirrhosis (choice C) because she tested negative for hepatitis C antibody. Unlike hepatitis B, the antibody to hepatitis C does not confer immunity. It marks the presence of an infection in most cases. The hepatitis C antibody test is extremely sensitive. A negative antibody test in the presence of cirrhosis is usually seen only in immunocompromised hosts.

Although the patient has a history of a positive PPD test many years ago, she does not have peritoneal tuberculosis (choice E). Patients with peritoneal TB almost always have an elevated leukocyte count, usually with lymphocyte predominance.

45
Q

A 56-year-old man presents to the emergency department with fever and abdominal pain. He has a past medical history that is significant for hypertension, constipation, and diverticulosis. Two days ago, he began to experience profound left lower quadrant pain and, over the next 48 hours, he had high fevers and nausea. His temperature is 38.3 C (101 F), and his abdomen is diffusely tender with marked direct tenderness in the left lower quadrant. Laboratory analysis shows a leukocyte count of 18,300/μm3, with 91% polymorphonuclear cells and 6% band forms. Which of the following procedures would be most likely to confirm the likely diagnosis?

(A) Abdominal CT scan
(B) Abdominal ultrasound
(C) Flat and upright abdominal radiographs
(D) Barium enema
(E) Exploratory laparotomy

A

Respuesta: A

The correct answer is A. This patient has diverticulitis. This disorder classically presents with fever, left lower quadrant pain, and an elevated white count with a left shift. Most diverticula are right- sided, but most ruptured diverticula are on the left. Diverticular disease is associated with constipation and is a significant cause of lower gastrointestinal bleeding. With this patient, the physical and laboratory findings are highly suggestive of diverticulitis, but an abdominal CT scan has the required sensitivity to detect diverticula, as well as any possible abscess formation.

Abdominal ultrasound (choice B) has no role in diagnosing diverticular disease. Ultrasound is useful for detection of masses, stones, and gross changes in organ size or anatomy. Its sensitivity in detecting small diverticula is less than 10%.

Flat and upright abdominal radiographs (choice C) are not useful tests to diagnose diverticular disease, but are very useful in the ruling out intra-abdominal free air from perforation and small or large bowel obstruction.

Barium enema (choice D) at one time was the preferred method for diagnosing diverticulitis, since the leakage of the barium from the ruptured diverticulum was easily visualized. However, a number of studies have suggested that such leakage is in fact harmful. Largely because of these observations, as well as the readily available nature of CT scanners at most institutions, CT scanning has become the imaging modality of choice for making the diagnosis of diverticulitis.

Exploratory laparotomy (choice E) is a major surgical procedure and, as such, is limited to cases in which there is such a high suspicion of massive abdominal pathology that even the high sensitivity of CT scan (>95%) is not adequate. An example of such a case would be perforating abdominal trauma.

46
Q

A 58-year-old woman comes to the physician because of persistent joint aches affecting hands and hips in an asymmetric distribution. The pain is slow in onset and is aggravated by activity. She reports a brief (less than 30 minutes) phase of morning stiffness relieved by heat and movement. She denies fever or weight loss. On the contrary, she has gained approximately 5% of her baseline weight in the past 6 months. Her temperature is 37 C (98.6 F), blood pressure is 130/80 mm Hg, pulse is 74/min, and respirations are 12/min. Examination reveals nodular thickening of the distal interphalangeal joints without redness. Mild limitation in joint motion is appreciated bilaterally in hand joints and in the right hip joint. The patient’s walking is characterized by a slightly shortened length of stride on the right side. Cardiac and respiratory examination reveals no abnormalities. At this time, which of the following is the most appropriate next step in diagnosis?

(A) No further evaluation necessary
(B) Complete blood count and erythrocyte sedimentation rate
(C) Blood test for rheumatoid factor
(D) Blood test for antinuclear antibodies
(E) X-ray studies
(F) Bone densitometry
(G) Diagnostic arthrocentesis

A

Respuesta: A

The correct answer is A. In the presence of this classic symptomatology, the clinical judgment alone is sufficiently accurate. Thus, no further laboratory or radiologic investigations are needed to support a diagnosis of primary osteoarthritis in a patient with characteristic signs and symptoms. If there are atypical manifestations, further investigations may be indicated to rule out other conditions.

Complete blood count and erythrocyte sedimentation rate (choice B) are not needed in this case, but may be occasionally helpful in excluding inflammatory causes of joint diseases.

A blood test for rheumatoid factor (choice C) would be indicated in the presence of signs and symptoms suggesting rheumatoid arthritis. Symmetric involvement of small joints, associated with low-grade fever, fatigue, and prolonged stiffness, suggests the need for rheumatoid factor testing.

A blood test for antinuclear antibodies (choice D) is useful in ruling out collagen vascular diseases. Among these, systemic lupus erythematosus may present with polyarticular inflammation, but this is usually associated with multiorgan involvement as well as constitutional symptoms.

X-ray studies (choice E) are not necessary in the diagnostic assessment of typical osteoarthritis. Radiologic signs of degenerative osteoarthritis include narrowing of joint space, osteophytes, subchondral sclerosis, and intraarticular bone fragments (“joint mice”).

Bone densitometry (choice F) is indicated in patients with osteoporosis, as a part of diagnostic evaluation or during treatment follow-up.

Diagnostic arthrocentesis (choice G) allows microscopic examination of synovial fluid. With this procedure, the nature of the fluid (transudate or exudate) can be determined, microorganisms identified by Gram staining and cultured, and crystals evaluated by polarized microscopy. It is rarely necessary in degenerative osteoarthritis.

47
Q

A 55-year-old man has been known for many years to have cirrhosis of the liver secondary to hepatitis C. His clinical condition had been stable until approximately 3 months ago, when he seemed to decompensate. He has developed worsening jaundice, increased ascites, and mild encephalopathy. For the past 3 weeks he has also complained of vague, constant, right upper quadrant abdominal pain. Physical examination shows a nodular liver, but it is no different than it had been in the past. CT scan demonstrates the presence of a solid tumor mass near the dome of the right lobe, where it could not be felt by palpation. The mass is approximately 8 cm in diameter, and it was not seen on a CT scan that had been done a year earlier. Because of the location of the mass, the radiologist is reluctant to attempt a needle biopsy. Which of the following is an additional useful diagnostic test in this patient?

(A) Alpha-fetoprotein (AFP)
(B) Carcinoembryogenic agent (CEA)
(C) 5 hydroxy-indoleacetic acid (5HIAA)
(D) Hepatitis C titers
(E) Portal vein angiogram

A

Respuesta: A

The correct answer is A. AFP is the blood marker for hepatocellular carcinoma, the tumor most likely to be present in this man. Although moderate elevations of the marker will occur just because of the cirrhosis, higher levels are virtually diagnostic for the tumor.

CEA (choice B) is the marker for metastatic colon cancer. Had the background been the discovery of a liver mass in a patient who previously had had colorectal cancer resected, this would have been the correct answer.

5HIAA (choice C) is diagnostic for the carcinoid syndrome. Liver masses also would be a feature in that condition, as patients do not develop the syndrome unless they have liver metastasis. The clinical picture would have included episodes of flushing of the face, diarrhea, and bronchoconstriction. Long-term damage to the right side heart valves might also have been present.

This man will have elevated titers for hepatitis C (choice D), and the levels will not be diagnostic for the development of cancer.

Should a surgical resection be planned, arteriograms might be done. Studies of the portal vein (choice E) would be less likely to be undertaken, and if done they would add only information about location and spread, rather than about the nature of the tumor.

48
Q

Select the most likely mode of transmission:

A 28-year-old G2P1 woman comes to the physician at 30 weeks gestation with abdominal pain, cramps, and passage of clear vaginal fluid. The patient undergoes premature labor and delivers a stillborn child. Her only recent history is a flu-like illness with fever, myalgias, and mild diarrhea.

(A) Coyote bite
(B) Exposure to cows or sheep
(C) Exposure to ticks
(D) History of contact with rabbits or other rodents
(E) History of recent travel to tropical countries
(F) Ingestion of home-canned food
(G) Ingestion of inadequately cooked pork
(H) Ingestion of inadequately cooked salmon
(I) Ingestion of unpasteurized milk products
(J) Prior transfusion of blood or blood products
(K) Raccoon bite
(L) Recent admission to a hospital
(M) Recent exposure to a child with chickenpox
(N) Unprotected sexual intercourse
(O) Use of IV drugs
(P) Working in a hemodialysis center
(Q) Working in an aviary

A

Respuesta: I

The correct answer is I. The clinical presentation is that of listeriosis caused by Listeria monocytogenes. Patients at risk include pregnant women, the elderly, newborns, and immunodeficient patients after eating contaminated foods. Listeria is found in unpasteurized milk, delicatessen meats, cheese, and raw vegetables. In pregnant women, it can cause miscarriages, premature labor, and stillbirths. Half of all newborns who are infected with Listeria die from the illness.

49
Q

Select the most likely mode of transmission:

A 45-year-old man comes to the physician 4 days after developing a pruritic papule on his right forearm. This lesion transformed into a vesicle and then a black eschar within 36 hours of onset. On examination, the eschar is surrounded by extensive edema and erythema and 4-5 peripheral vesicles. He has been afebrile all this time, but now his temperature is 39.0 C (102.2 F). Smears of the skin lesion reveal gram-positive encapsulated rods.

(A) Coyote bite
(B) Exposure to cows or sheep
(C) Exposure to ticks
(D) History of contact with rabbits or other rodents
(E) History of recent travel to tropical countries
(F) Ingestion of home-canned food
(G) Ingestion of inadequately cooked pork
(H) Ingestion of inadequately cooked salmon
(I) Ingestion of unpasteurized milk products
(J) Prior transfusion of blood or blood products
(K) Raccoon bite
(L) Recent admission to a hospital
(M) Recent exposure to a child with chickenpox
(N) Unprotected sexual intercourse
(O) Use of IV drugs
(P) Working in a hemodialysis center
(Q) Working in an aviary

A

Respuesta: B

The correct answer is B. The clinical presentation is that of anthrax, which is caused by Bacillus anthracis. Anthrax is a disease of sheep, cattle (choice B), horses, goats, and pigs, but the agent can be transmitted to humans through broken skin or mucous membranes, or through inhalation. This infection is now rare in the U.S. and is seen only in persons exposed to infected animals or animal products. Thus, farmers, veterinarians, and workers in the wool industry are susceptible to anthrax. Penicillin G is the treatment of choice.

Coyote bite (choice A) and raccoon bite (choice K) may expose the victim to the risk of rabies. In fact, several wild animals may transmit rabies virus. In the U.S., potentially rabid animals include raccoons in the East and New England, skunks in the West and Midwest, coyotes in Texas, and foxes in the Southwest, New England, and Alaska. Contrary to popular belief, rodents are very unlikely to have rabies.

Exposure to ticks (choice C) may result in a number of infective illnesses, which depend on the regional distribution of the ticks and organisms involved. The following is a list of infections known to be transmitted to humans by ticks: arbovirus encephalitis, babesiosis, Colorado tick fever, ehrlichiosis, hemorrhagic fever, Lyme disease, relapsing fever, Rocky Mountain spotted fever, and tularemia.

History of contact with rabbits or other rodents (choice D) may be found in humans affected by tularemia, caused by Francisella tularensis. It affects wild rodents, especially rabbits and muskrats. The disease may be acquired by contact with infected tissue (such as skinning rabbits) or from tick bites.

History of recent travel to tropical countries (choice E) may be linked to a great variety of exotic infectious processes, from innocuous and self-limiting, such as traveler’s diarrhea, to potentially life-threatening, such as malaria. Suffice it to say that travel history should always be elicited when investigating the etiology of infectious diseases.

Ingestion of home-canned food (choice F) is typically associated with botulism, caused by Clostridium botulinum. This is a ubiquitous, strictly anaerobic, spore-forming bacillus that finds excellent growth conditions in the anaerobic environment of home- canned or vacuum-packed foods.

Ingestion of inadequately cooked pork (choice G) may result in parasitic diseases, such as cysticercosis (due to Taenia solium), trichinosis (Trichinella spiralis), and toxoplasmosis (Toxoplasma gondii).

Ingestion of inadequately cooked salmon (choice H) may result in tapeworm infection due to Diphyllo-bothrium latum. Cases of fish tapeworm are observed in the Pacific Coast of the U.S. and many other temperate and subarctic lake regions in the world.

Ingestion of unpasteurized milk products (choice I) has become a very rare mode of transmission of infections. It was once commonly responsible for gastrointestinal tuberculosis and brucellosis.

Recent admission to a hospital (choice L) is a common predisposing condition for severe hospital-acquired infections, the most common of which are urinary tract infection and pneumonia. Hospital-acquired infections have become a major health problem, especially because hospital-acquired organisms are often resistant to most antibiotics.

Recent exposure to a child with chickenpox (choice M) exposes non-immune individuals to chickenpox, but also to shingles.

Transfusion of blood and blood products (choice J), sexual contact (choice N), accidental needlesticks among health professionals and in dialysis units (choice P), and intravenous drug use (choice O) are all related to viral hepatitis transmission.

Working in an aviary (choice Q) is a risk factor for psittacosis, an atypical pneumonia caused by Chlamydia psittaci. Transmission is via dust from dry bird secretions and feces. Patients present with fever, nonproductive cough, myalgias, and x-rays that appear more severe than clinical presentations.